pdf

School

Rutgers University *

*We aren’t endorsed by this school

Course

203

Subject

Chemistry

Date

Jan 9, 2024

Type

pdf

Pages

93

Uploaded by SargentSeaLionMaster1039

Report
General Chemistry 11 material begins here Sample datasheet and periodic table provided as reference on the next page. 111
Periodic Table and sample datasheet AEC.S Exams INSTANTS & ABBREVIATIONS AND SYMBOLS I amount of substance | gas constant R | molar mass M o S atmosphere atm | gram g |mole mol R=8314Jmol"K atomic mass unit u | hour h | Planck’s constant /| | R=0.0821 L-atm-mol-K™" atomic molar mass A | joule J | pressure 2 Na = 6.022 x 105 mol™! Avogadro constant N | kelvin K | second s h=6.626x 103 I's Celsius temperature °C | kilo— prefix k | speed of light c 2 ; centi— prefix c | liter L |temperature, K T €=2.998 x 10°m-s” energy of activation E, | measure of pressure mmHg | time t 0°C=273.15K efir;fll::}; 1~\II ::)Ill; prefix 1\‘2 volume v 1 atm = 760 mmHg s 1 atm = 760 torr EQUATIONS Equations may be included in this position of your exam. These vary depending on the type of exam you are taking. For example, if you are expected to do a calculation involving an integrated rate law, you may be provided with a series of equations in this position. PERIODIC TABLE OF THE ELEMENTS 1 1 H 1,008 2 137145715116+ 17+ sio0s. 3 4 5 6 7 8 9 Li | Be B Cil N .| O K, 6941 | 9.012 1081 | 1201 | 1401 | 1600 | 1900 | 2018 11 12 131415 )16 | 17 64 | 65 | 66 | 67 | 68 | 69 [ 70 | 71 Ce | Pr (Nd |Pm [Sm | Eu | Gd | Tb | Dy [ Ho | Er [ Tm | Yb | Lu 140.1 | 1409 | 1442 1504 | 1520 | 1573 | 1589 | 1625 | 1649 | 1673 | 1689 | 1730 | 1750 90 | 91 | 92| 93|94 (959 |97 |98 |99 [100]101]102]103 Th | Pa | U | Np | Pu|Am (Cm | Bk | Cf | Es | Fm [ Md | No | Lr 2320 | 2310 | 2380 Please note that the periodic table changes to keep current with recent discoveries. The periodic table you use may vary from the table shown here. 112
Chapter 9 Solutions and Aqueous Reactions, Part 2 Chapter Summary: This chapter will focus on the additional aspects of solutions, usually found in the second semester of a standard general chemistry course (whereas the first part of solutions is usually found in the first semester). Concepts discussed here and explained in example items go into more detail related to intermolecular forces associated with solution formation and properties of solutions. Reactions of solutions are discussed separately in chapters related to equilibrium. Specific topics covered in this chapter are: Solution formation including intermolecular forces in solutions Solubility as it relates to intermolecular forces Concentration units and conversions between Solubility of solids and gases, particularly in aqueous solution Colligative properties including freezing point depression, boiling point elevation and osmotic pressure Previous material that is relevant to your understanding of questions in this chapter include: The mole and formula calculations (Chapter 3) Stoichiometry and balancing equations (Chapter 4) Molar concentrations and solution stoichiometry (Chapter 5) Lewis dot structures and polarity (Chapter 7) Intermolecular forces (Chapter 8) Common representations used in questions related to this material: Name Example Used in questions related to Structures Intermolecular forces in solutions Temperature dependence of Graphs solubility of solids 0 10 20 30 40 50 60 70 80 90 Temperature, °C Where to find this in your textbook: The material in this chapter typically aligns to a solutions chapter (could be labeled as “Physical Properties of Solutions™) in your textbook. The name of your chapter may vary. Practice exam: There are practice exam questions aligned to the material in this chapter. Because there are a limited number of questions on the practice exam, a review of the breadth of the material in this chapter is advised in preparation for your exam. How this fits into the big picture: The material in this chapter aligns to Intermolecular Interactions (4) as listed on page 12 of this study guide. 13
Your preview ends here
Eager to read complete document? Join bartleby learn and gain access to the full version
  • Access to all documents
  • Unlimited textbook solutions
  • 24/7 expert homework help
Solutions and Aqueous Reactions, Part 2 cACS w Exams Study Questions (SQ) SQ-1. Where is the hydrophilic (attracted to water) region Region I Region 2 Region 3 of the molecule? HH H HeR] (A) Region | \/ I (B) Region2 N R (C) Region3 / | / (D) The three regions are equally hydrophilic n i o Knowledge Required: (1) Structure of water. (2) Intermolecular forces (hydrogen bonding). Thinking it Through: As you read this question. you realize the first step will require that you consider the structure of water in relation to the structure of the molecule because you are asked to consider how water and the shown molecule will interact. Continuing along these lines, you then compare the molecules and consider how these will interact within the context of hydrogen bonding. You know that in order to have two molecules form a hydrogen bond, you will need hydrogen bonded to N, O, or F and a lone pair of electrons on N, O, or F. In a pure substance, these will need to be present in one molecule (as all molecules are the same in a pure substance). Ina solution, molecules could have only part of these requirements with the other molecules having the other components. Therefore, you draw water molecules in relation to the molecule, including the relative partial charges (to help make the point of attraction): & hydmgenbonding% H/ \H Therefore, water is attracted to region 3 (Choice (C)) and not region 1 (Choice (A)) or region 2 (Choice (B)) or all three equally (Choice (D)). NOTE: Drawing your own representations is often an effective problem-solving strategy in chemistry. Practice Questions Related to This: PQ-1 and PQ-2 SQ-2. ‘Which molecule is most soluble in water? o~ o o @ ! ® g PN CH, CH, HiC CHy [o] © l o A e O P HeC CHS CHY HaC OH 114
.>ACS Solutions and Aqueous Reactions, Part 2 W Exams Knowledge Required: (1) Structure of water. (2) Intermolecular forces (hydrogen bonding). Thinking it Through: Similar to the previous study question, to solve this you are going to consider each structure interacting with the structure of water, paying close attention to components that would support greater attraction due to hydrogen bonding. As a reminder again, in a solution these are: 1. Hydrogen bonded to N, O, or F 2. Alone pair of electrons on N, O, or F Molecules interacting with water can have one or more locations for this attraction to occur. Considering each molecule with water, including relevant partial charges and hydrogen bonding: . 0. W hydrogen & bonding <33 hydrogen & —eeeeT bonding@ HsC e} CHy N N cnz/ CH,/ no hydrogen bonding & CH, CH; CH; O A Z\CH/ A H/"\H 2 B From these diagrams, you see that the greater the number of attractions, the more soluble the molecule will be in water. Adding to this decision, you consider the relative sizes of the molecules (as solubility can be conceptualized as a “substitution” process of the solute molecules into the water molecule’s location, thus needing the same relative attractions and size). You see that the molecules are reasonably the same size. Therefore, you can see that one molecule will not hydrogen bond with water at all (Choice (D)), therefore this molecule will be the least soluble in water. You also see that molecules in Choices (A) and (B) have limited attractions to water making these incorrect as well. Choice (C) is ideal for substitution, as it has multiple opportunities for hydrogen bonding, making (C) or acetic acid, the correct choice. Practice Questions Related to This: PQ-3. PQ-4, and PQ-5 SQ-3. A solution of NaCl in water has a concentration of 20.5% by mass. Molar mass / g'mol™ ‘What is the molal concentration of the solution? NaCl 5844 (A) 0.205m (B) 0258m ©) 35im D) 441m Knowledge Required: (1) Concentration unit of molality. (2) Concentration unit of percent by mass. Thinking it Through: You see this question starts by giving you percent by mass and asks you to determine molal concentration. So, to solve this, you first define both concentration units. First for percent by mass, you define this with a general definition, then for a solution and finally further define the mass of the solution as the sum of the masses of the solvent and solute: mass of one component ©100% = mass solute ©100% = mass solute %100% Percent by mass = " total mass mass solution mass solute + mass solvent mol solute Now you define molal concentration: ‘molal concentration = —————— mass solvent (kg) 115
Solutions and Aqueous Reactions, Part 2 . ACS Exams Before you set up your solution to the problem, you first map out wfiat;ofi will do: Set your Use the percent by Use molar Use the mass of Caleulate sample size _y | mass of thesolute to | _, mass to _y | solute/mass of solution | _ the molal ©0100g give the mass of the determine 1o determine the mass of conc solution solute solute solvent Now setup and solve the problem: NaCl (20.5¢ NaCl)[ i‘il—ac—) 100.0 g solution —20.5 g NaCl = 79.5 g water = w 441m e o€ 0.0795 kg water J . ‘Which is choice (D). Choice (C) uses the mass of the solution (in kg) rather than the mass of the solvent (water). Choice (B) omits calculating moles and use the mass of water in grams instead of kilograms. Choice (A) also omits calculating moles and use the mass of the solution (and not the solvent or water) in grams instead of kilograms. Practice Questions Related to This: PQ-6, PQ-7, PQ-8, PQ-9, and PQ-10 SQ-4. ‘What is the mole fraction of water in 200. g of 89% (by mass) ethanol, Molar mass / g'mol™! CoHsOH? C>HsOH 46 @A) o011 (B) 024 © 032 D) 076 Knowledge Required: (1) Concentration unit of mole fraction. (2) Concentration unit of percent by mass. Thinking it Through: Similar to the previous study question, you see this question starts by giving you percent by mass and asks you to determine mole fraction. So, again to solve this, you first define both concentration units: ‘mass solute Percent by mass = ——————————x100% mole fraction (X, )= mol solute 'mass solute + mass solvent ol mol solute + mol solvent Before you set up your solution to the problem, you first map out what you will do: Set your sample size to 100 g solution, Use molar mass Use molar mass Calculate making: mass.owe =% by massand | = | (solute)to | = | (solventyto | —> | the mole masSaiven = 100 % by mass determine nsoue determine riven fraction Now setup and solve the problem (norice the 200. g sample size is not needed): 100 g solution 89 g C,H,OH =11 g water (89 ¢ CZH,OH)[M] =1.9 mol C,H,0H 46 ¢ C,H,0H mol H,0 0.61 mol H,0 1 g0 MO ot notto x,, ——OSLmOIHO o, (11eH, )(mgHZo,‘ o 9~ 1.9 mol C,H,0H + 0.61 mol H,0 Which is choice (B). Choice (D) is the mole fraction of ethanol. Choice (A) is the percent by mass of water (as a fraction). Choice (C) uses the number of moles of solvent (ethanol) only and not the total number of moles. Practice Questions Related to This: PQ-6, PQ-7, PQ-8, PQ-9, and PQ-10 116
Your preview ends here
Eager to read complete document? Join bartleby learn and gain access to the full version
  • Access to all documents
  • Unlimited textbook solutions
  • 24/7 expert homework help
“ACS Solutions and Aqueous Reactions, Part 2 ¥ Exams SQ-5. A mixture of 100 g of K2Cr:07 and 200 g of M water is stirred at 60 °C until no more of the salt dissolves. The resulting solution is poured off into a separate beaker, leaving the undissolved solid behind. The solution is now cooled to 20 °C. What mass of K>Cr,0; crystallizes from the solution during the cooling? A) (B) 18¢g 01002030 40 50 60 70 80 90 © 3lg Temperature, °C Solubilify KCr0./ 100 ¢ ( D) 62g Knowledge Required: (1) Interpretation of a solubility graph. (2) Definition of solubility. Thinking it Through: The problems provides you with both quantitative data and an associated process. You think it would be easiest to map out what is described before coordinating this with the graph: 60°C 60°C 20°C You start with: Solution is poured off (saturated | _,| Cooled: some mass of K2Cr207 100 g K2Cr207 in 200 g water solution of K2Cr207 in 200 g water’ crystallizes out | N leaving undissolved solid Now you coordinate this with the graph, marking the solubility of K»Cr:0; at 60 °C and 20 °C: You see that: at 60 °C, the solubility is 40 g/100 g water / 50 the 60 °C solution above (once poured off) contained 80 g of K>Cr:0- in 200 g of water. at 20 °C, the solubility is 9 g/100 g water so the 20 ° C solution above (once cooled) contained 18 g of K2Cry07 in 200 g of water in solution. Meaning, the cooled solution had 80 g 18 gor 62 g of K»Cr,0; as a solid or crystallized from the water, 0 10 20 30 40 50 60 70 BO 90 which is choice (D). Temperature, "C Choice (B) is incorrect because this is the mass that remained in solution. Choice (C) is incorrect because this is the mass that crystallized out in 100 g of water, rather than 200 g with Choice (A) as the incorrect choice when considering 100 g of water and the mass that remained in solution. Practice Questions Related to This: PQ-11 and PQ-12 SQ-6. Carbonated beverages have a fizz because of gas dissolved in the solution. What increases the a‘fi,\d concentration of gas in a solution? (A) cooling the solution (B) heating the solution (C) increasing the volume of solution (D) decreasing the volume of solution Thinking it Through: Y ou recall that solubility of gases can be expressed quantitatively using Henry’s law which expresses relationship of the concentration of the gas (in the solution, ¢), the pressure of the gas above the solution (P), and a constant for the gas (k): ¢ = kP (also written as ¢ = Ku/P). From this, you see that as pressure of the gas 117
Solutions and Aqueous Reactions, Part 2 ."ACS W Exams increases, the concentration of the gas increases. You also suspect that solubility of gases vary based on their structure, and this would be expressed in the Henry’s law constant, k&. However, from this you also see that the volume of the solution will not change the concentration of the gas in the solution, so neither choice (C) or (D) are correct. You do know that the Henry’s law constant is a constant for a gas at a given temperature and as temperature increases, solubility of the gas decreases or k decreases. Therefore, heating the beverage will decrease the concentration of the gas (Choice (B)), but cooling the beverage will increase the concentration or fizziness, meaning Choice (A) is correct. Practice Questions Related to This: PQ-13 and PQ-14 SQ-7. When 100. g of an unknown compound was dissolved in 1.00 kg of water, the freezing point was lowered by 6.36 °C. What is the identity of this unknown compound? (K for water = 1.86°C-m™') (A) CsCl (B) KCI (©) LiF (D) NaCl Knowledge Required: (1) Colligative properties, freezing point depression. (2) Ionic éorfiponnds and van’t Hoff coefficients. (2) Molal concentration and molar mass. Thinking it Through: As you read the question, you approach this problem by first evaluating the equation for freezing point depression (which you know is the appropriate colligative property because of the cue to this in the question): AT = K,.m. You further examine the responses and see that all of the substances are ionic with the same 1:1 ratio of cation to anion, meaning that 2 ions form for each formula unit of the compound. Therefore, you refine the equation for freezing point depression for the van’t Hoff coefficient of 2 (i = 2): AT =iK;m=2K.m: _AT 6.36°C K, 2(1.86°C-m”) From here, you consider molal concentration and the other information provided in the problem to determine first the number of moles of solute and then (using the mass of solute), the molar mass of the solute: AT, =iK;m m =17lm molal concentration =~ SolUte_ L71mol |1 00 kg water) =171 mol solute mass solvent kg solvent 100. g solut A (&) =58.5g-mol ' which is NaCl or response (D). 171 mol solute Choice (A) would likely be selected if the calculation of molal concentration was inverted (calculated molar mass of 170 g'mol™'). Choice (B) may be selected if number of moles are calculated incorrectly by using the mass of solution rather than mass of solvent. Choice (C) may be selected if the van’t Hoff coefficient was omitted (calculated molar mass of 29 g'mol ™). Practice Questions Related to This: PQ-15, PQ-16, and PQ-17 SQ-8. Which 0.1 molal aqueous solution will have the lowest freezing point? M‘ (A) AI(NO3); (B) CaCl, (C) CHsOH (D) NaCl Knowledge Required: (i ) Collig’ativ’e fimfieniés; ffeéiing }S{)int dépression. (i) Tonic conipourids“and van’t Hoff | coefficients. Thinking it Through: Y ou know that colligative properties depend on the number of particles in solution (the molal concentration) but not the identity of the solute. You also know this is reflected in the equation for freezing point depression as noted in the previous study question. Because the concentrations are all the same (0.1 m) and the solvent is the same (water with the same freezing point depression constant, K7), the only component that will vary will be the van’t Hoff coefficient, i. So, you start by evaluating the number of particles in solution for each solute: 118
*ACS Solutions and Aqueous Reactions, Part 2 V Exams hydratic hydration equation deale AI(NOs)s(s) = Al**(aq) + 3NOs™(aq) CaCly(s) Ca**(aq) + 2CI(aq) 3 C2Hs0H(l) = C,HsOH(aq) NaCl(s) - Na*(aq) + Cl-(aq) 2 Note: icsic is used here because you don’t have the measured value (imess); in calculations, imcss Would be better than icalc. You are asked for the lowest freezing point or largest ATr. Therefore, choice (A) is correct with a caleulated van’t Hoff coefficient of 4. The other choices are incorrect because the coefficients are all smaller than the value for AI(NOs)s. Practice Questions Related to This: PQ-18, PQ-19, and PQ-20 Practice Questions (PQ) M} PQ-1. Which pair of compounds will form hydrogen bonds with one another? (A) CHsand H.O (B) CH:and NH; (C) HF and CHs4 (D) H.0 and NH; w‘"’”’\ PQ-2. Which pair of compounds will form hydrogen bonds with one another? cl H HC, CH; @) : \\0th \2 and H—Cl l and l H,C™™ 2™~CH, // H// \ H [ . T | ©) C. and C. D) and C. w7 Son w7 // "™ H H H PQ-3. What would be expected to be the most soluble in ethanol (CH;:CH,OH)? (A) CH;CH.CH3 (B) CO: (C) NH; (D) SFe M PQ-4. Which substance is most soluble in water? (A) ethane, CH;CH3 (B) ethanol, CH;CH,OH (C) n-butane, CH;CH,CH>CH; (D) 1-butanol, CH;CH,CH>CH,OH PQ-5. Consider the structure of Vitamin A; CHs CH; CHs (C20H300). Vitamin A, should be | " | H ; AN NN NN (A) insoluble in water and in oil. e SN N e N N N (B) less soluble in water than in oil. H,l l N, \\cnX (C) equally soluble in water and in oil. i Chy (D) more soluble in water than in oil. PQ-6. The mass percent of acetic acid (CH;COOH) in a bottle of vinegar is Molar mass / g'mol™! 5.45% in water. What is the molar concentration of acetic acid in CH;COOH 60.06 vinegar, assuming the density of vinegar is 1.005 g-mL™'? (A) 0.545M (B) 0.903M (C) 0.908 M (D) 0912M 119
Your preview ends here
Eager to read complete document? Join bartleby learn and gain access to the full version
  • Access to all documents
  • Unlimited textbook solutions
  • 24/7 expert homework help
Solutions and Aqueous Reactions, Part 2 . ACS & Exams PQ-7. A 250-mL bottle of a sports drink solution contains 4.50% by mass of Molar mass / g sodium chloride. What is the molal concentration of sodium chloride in NaCl . this bottle of sports drink? (A) 0.806m (B) 0.770m (©) 0.180m (D) 0.0113m PQ-8. A 1400 g sample of stream water contains 12.2 ppm of mercury. What is the mass of Hg in the sample? (A) 0.11mg (B) 2.4mg (C) 8.7mg (D) 17 mg PQ-9. What mass of water is needed to dissolve 292.5 g of NaCl to produce a Molar mass / g-mol™! 0.25 m aqueous solution? NaCl 58.44 (A) 20kg (B) 5.0kg (C) 0.80kg (D) 0.050 kg PQ-10. The density of a 3.539 M HNO; aqueous solution is 1.150 g-mL " at Molar mass / 20°C. What is the molal concentration? HNO;3 (A) 3.077m (B) 3818m (C) 3.946m (D) 5252m PQ-11. The solubility of a substance is 60 g per 100 mL of water at 15 °C. A solution of this substance is prepared by dissolving 75 g in 100 mL of water at 75 °C. The solution is then cooled slowly to 15 °C without any solid separating. The solution is. (A) supersaturated at 75°C. (B) supersaturated at 15°C. (C) unsaturated at 15°C. (D) saturated at 15°C. PQ-12. A student mixes 20.0 g of a saltin 100.0 g of Solubility at 20 °C in 100.0 g of water water at 20 °C and obtains a homogeneous KCl 100¢g solution. Which salt could this be and why? KNO; 300g (A) KCl because it is saturated. (B) KClI because it is unsaturated. (€) KNO; because it is saturated. (D) KNO:; because it is unsaturated. PQ-13. The solubility of carbon dioxide in water is very low in air (1.05x10°5 M at 25 °C) because the partial pressure of carbon dioxide in air is only 0.00030 atm. What partial pressure of carbon dioxide is needed to dissolve 100.0 mg of carbon dioxide in 1.00 L of water? (A) 0.0649 atm (B) 2.86atm (C) 28.6atm (D) 64.9 atm Up"'vp\ PQ-14. How will you increase the solubility of oxygen in water? The partial pressure of oxygen ( B, )is 0.21 atm in air at | atm (Pex). (A) increase P, but keep Pey constant (B) decrease £, but keep Py constant (C) increase Pey but keep B, constant (D) decrease Pe but keep £, constant PQ-15. A 2.50 g sample of naphthalene, CioHs, was dissolved in 100 g of Molar mass / g'mol™! benzene. What is the freezing point of the benzene solution? The freezing CioHs 128.17 point of pure benzene is 5.45 °C; Kr=5.07 °C-m™". (A) ~0.989°C (B) 0.989°C (C) 446°C (D) 6.44°C 120
g cACS Solutions and Aqueous Reactions, Part 2 Exams PQ-16. A technician has the measured osmotic pressure of a solution to I Temperature determine the molar mass of a covalent solute. Which other 1L Volume of solution information would need to be measured in order to determine the 1L Mass of solute molar mass? (A) OnlyI (B) Only I (C) OnlyIand IT (D) LI, and I PQ-17. A solution was made by adding 800 g of ethanol, C;HsOH, to 8.0x10° g Ki/°C-m! of water. How much would this lower the freezing point H.0 1.86 (A) 32°C (B) 4.1°C (©) 82°C (D) 16°C M PQ-18. Which aqueous solution will have the lowest osmotic pressure? (A) 0.04 M MgCl, (B) 0.05M KBr (©) 0.1 MLiCI (D) 0.2 M Na:SOs PQ-19. Consider pure water separated from an aqueous sugar solution by a semipermeable membrane. After some time has passed, what (if anything) will happen to the concentration of the sugar solution? (A) It will decrease. (B) It will increase. (C) Ttwill remain the same. (D) It cannot be determined. PQ-20. Which solution has the | Solution: Molar mass / g-mol™' higher boiling point? | (1) 200.0 g glucose dissolved in 1.00 kg of water glucose, 180 (2) 200.0 g sucrose dissolved in 1.00 kg of water sucrose, 342 (A) solution 1 (B) solution 2 (C) Both would boil at the same temperature as pure water. (D) Both would boil at the same temperature, but above that of pure water. 121
Solutions and Aqueous Reactions, Part 2 = ACS Y Exams Answers to Study Questions 1. C 5. D 2. C 6. A 3 D 7. D 4. B 8 A Answers to Practice Questions 1. D 11. B 22 D 12. D 3. C 13. A 4. B 4. A 5. B 15. C 6. D 16. D 7. A 17. B 8 D 18. B 9. A 19. A 10. B 20. A 122
Your preview ends here
Eager to read complete document? Join bartleby learn and gain access to the full version
  • Access to all documents
  • Unlimited textbook solutions
  • 24/7 expert homework help
Chapter 10 Kinetics Chapter Summary: This chapter will focus on chemical kinetics. This includes the topics of reaction rate and rate laws. Methods of determining the rate laws will be presented and calculations with integrated rate laws will be reviewed. The collision theory of chemical kinetics and reaction mechanisms will be reviewed. Specific topics covered in this chapter are: Determining a rate law from plots of data Method of initial rates Solving problems using integrated rate laws Half-life problems Collision theory and reaction energy diagrams Calculation of activation energy for a reaction Reaction mechanisms Relationship of kinetics to equilibrium Previous material that is relevant to your understanding of questions in this chapter include: Significant figures (Toolbox) * Scientific notation (Toolbox) Equilibrium (Chapter 11) « Enthalpy (Chapter 6) Common representations used in questions related to this material: Name Example Used in questions related to Compound units M-s'; L-mol™"s ! rates and rate constants Energy diagrams activation energy Where to find this in your textbook: The material in this chapter typically aligns to “Kinetics” or “Chemical Kinetics” in your textbook. The name of your chapter may vary. Practice exam: There are practice exam questions aligned to the material in this chapter. Because there are a limited number of questions on the practice exam, a review of the breadth of the material in this chapter is advised in preparation for your exam. How this fits into the big picture: The material in this chapter aligns to the Big Idea of Kinetics (7) as listed on page 12 of this study guide. 123
Kinetics = ACS —¥ Exams Study Questions (SQ) SQ-1. What is the rate law of this reaction? Trial# | [H2lo/M | [NOlo/M | Initial rate / M-s~ 1 LOx107 | 1.0%10° 2.0x10° 2Ha(g) + 2NO(2) Na(2) 2H:0(g 2 2.0x10° | 2.0x10° 1.60%10* “© @~ © 3 [ 20x107 [ 1ox107 4.0410° (A) rate = [H2[NO] (B) rate = K[Hy][NOJ* (€) rate = A[H.2[NOJ® (D) rate = &{Ny)[H0}%/[H:]*[NOJ | Knowledge Required: (1) Definition of rate law. (2) How to use the method of initial rates to determine the rate law. Thinking it Through: You are given data with varying starting amounts (initial concentration noted as the “0” subscript, i.e. [Hz]o) of each reactant and the reaction rate that was measured. You write the generic rate law for the reaction as: rate = A[H;NOJ To find the value of each exponent, you select a pair of trials that holds one concentration constant and allows you to see the effect on the rate by changing the other concentration. You select Trials 1 and 3 to determine the value of x. Taking the ratio of the trials you find: (NOTE: The & will cancel out.) raie, K{H,J'[NOJ rate, k[H,J'[NOJ 4.0x10" _ k(2.0x107)" (L0x107)" 9 (2.0x107)' (1.0x107)’ 20x10° k(LOX10 ) (LOx107)" (1.0x107) (1.0x107) x=1 The order of the reaction with respect Hy is 1. Repeating this process for Trials 2 and 3 to find the value of y. rate, _ k[H,T'[NOJ" rate; k[H,]'[NOJ" 1.6x10° k(20x107)'(2.0x10°)' . (2.6x107) (20107 4.0x10° k(20x107)y (1.0x10°)" (20x107)(1.0x107) y=2 The order of the reaction with respect NO is 2. The rate law for this reaction is rate = [H2)[NOJ. This is choice (B). Choice (A) is incorrect because it reverses the exponents. Choice (C) is incorrect because it uses the coefficients from the balanced equation. Choice (D) is incorrect because it is the equilibrium constant expression for the reaction. Practice Questions Related to This: PQ-1, PQ-2, and PQ-3 124
*ACS Kinetics SQ-2. The rate law for the reaction w.&“‘ A+B—C+D is first order in [A] and second order in [B]. If [A] is halved and [B] is doubled, the rate of the reaction will (A) remain the same. (B) be increased by a factor of 2. (©) be increased by a factor of 4. (D) be increased by a factor of 8. Knowledge Require&:'-(‘lj The meanings of the terms rate law, first order, and second order. (2) The ability to use the rate law expression. Thinking it Through: You are told the order of the reaction with respect to each reactant. You recognize that this is telling you the exponents in the rate expression. The exponent for [A] is 1 and the exponent for [B]is 2. You write the rate expression for the reaction: rate = K[A](B]? You realize the problem is asking you to determine the effect of changing the initial concentrations of both reactants on the rate of the reaction. This is a similar problem to SQ-1, where you were given the initial amounts and the rates but were asked to find the exponents. Here you know the exponents and the amounts and are being asked to find the ratio of the rates. As before you set this up as a ratio of the two rates (for two reaction trials noted by “17and “2” subscripts): Where rate:, [A]:, and [B]; are the rate, concentration of A, concentration of B for trial #1 and rate;, [A], and [B], are the rate, concentration of A, concentration of B for trial #2. The ratio of the rates is what you need to find. You also know the relationship between [A]; and [A] and [B]; and [B], from the information in the question: [A]2 =% [A]: (because “[A] is halved™) and [B]2 = 2[B]; (because “[B] is doubled™) Substituting these into the expression you get: e, _kALEE _A(J)CB1) k(Ja)4m)’ rate, k[A)[B] K[A),[BY K[A][B]} e, (1) The correct answer is choice (B). Choice (A) is incorrect because the 2 was not squared in the expression (2[B])2. Choice (C) is incorrect because it omitted the %2 term. Choice (D) is incorrect because the % term was mistakenly put in the denominator rather than the numerator. Practice Questions Related to This: PQ-4 and PQ-5 SQ-3. The half-life for the first order conversion of cyclobutene to ethylene, C:Hs(g) —2C2Ha(g) is 22.7 s at a particular temperature. How many seconds are needed for the partial pressure of cyclobutane to decrease from 100 mmHg to 10 mmHg? A) 0.101s ®) 5205 © 755s D) 52335 125
Your preview ends here
Eager to read complete document? Join bartleby learn and gain access to the full version
  • Access to all documents
  • Unlimited textbook solutions
  • 24/7 expert homework help
Kinetics . ACS v Exams Knowledge Required: (1) The relationship between half-life and the rate constant for first-order reaction. (2) The ability to use the half-life equation and the integrated first-law equation. Thinking it Through: You are given a reaction and told that it is first-order. You need to select and use the first- order integrated rate law to find the amount of time needed for the pressure to fall from 100 mmHg to 10 mmHg. You realize that you are not given a value for the rate constant, . You recognize that because you know the half- life, you can find the value of the rate constant. Because the reaction is first-order you use the half-life expression for a first-order reaction: _In2 k Rearranging and substituting the given half-life you calculate the value of k. In2 In2 B k=—= =0.0305s 22.7s You then use this value of k in the first-order intégrated rate law. In[C,H, ] =~k +In[C H,] ICHI-IICHL _ I0(10)~In(100) - which is choice (C). k 0.03055s" Choice (A) is incorrect because it used the value of the half-life as the rate constant. Choice (B) is incorrect because it used 1/half-life as the rate constant, instead of In2/half-life. Choice (D) is incorrect because it used the expression for the second order half-life to find the value of k. Practice Questions Related to This: PQ-6 and PQ-7 SQ-4. The half-life for the first-order radioactive decay of 2P is 14.2 days. How many days would be required for a sample of a radiopharmaceutical containing **P to decrease to 20.0% of its initial activity? (A) 33.0d (B) 49.2d ©) 71.0d (D) 286d Knowledge Required: (1) The knowledge that radioactive decay is first-order. (2) The ability to use the integrated form of the first-order rate law. (3) Relationship between half-life and rate constant. Thinking it Through: You are being asked to find the time for a process to occur. You know that you have to use the integrated rate law. You also know that nuclear decay is a first-order process. To use the first-order integrated rate law you need a value for the rate constant. You use the relationship for the first-order half-life: 2 k _ 2 2 " Solving for k you get: k = =0.0488 d 4 v 14.2d The question is asking you for the time required for the sample to decrease to 20.0% of its original activity. This is different from other problems that give you specific amounts of initial and final concentrations. To solve the problem, you realize that you can pick any initial amount and the final amount will be 20.0% of that initial value. To keep the math simple, you assume you started with 100.0%. You then need to find the time required for the amount to decrease to 20.0% of 100%, or 0.200%1.000 = 0.200. You now know: [Pl =1.00; [**P]=0.20; k=0.0488d ¢ You then use the integrated first-order rate law to find the time needed. 126
- ACS Kinetics W Exams In[“P]=~kr +In{*P), In[=P]— ]"[“P]u ~ In(0.200)~ In(1.00) _ 3304 which is choice (A). k 004884 ' Choice (B) is incorrect because it represents 5 half-lives multiplied by In(2). Choice (C) is incorrect because it assumes it took 5 half-lives to get to 20.0% left. Choice (D) is incorrect because it is 20 half-lives. Practice Questions Related to This: PQ-8 and PQ-9 SQ-5. Plots are shown for the reaction NOx(g) NO(g) + 2 Oa(g). What is the rate law for the reaction? = 5 g time time time k B) te =k ! Al te = rate =k x (A) rate (B) NO.) (€) rate=k[NO,] (D) rate = k[NO, |’ Knowledge Required: (1) Definition of rate law. (2) Ability to interpret integrated rate law equations. Thinking it Through: You are given three plots of the concentration versus time data for a reaction. Each plot differs in how it plots the concentration of NO,. You recall the three common integrated rate law expressions for reactions with a single reactant: X P Order Rate Law Integrated Rate Law Linear Plot zero rate = k [X]e=—kt + [X]o [X] vs time first rate = k[X]' In[X], = —¢ + In[X], In[X] vs time second rate = K[XJ* V[X]= ke + 1/[X]o 1/[X] vs time Each of the integrated rate laws has the form of a y = mx + b equation. When you examine the plots provided, you notice that the plot of 1/[NO;] vs time is linear and the others are nonlinear. You decide that this means the rate law is a second order rate law: rate = k[NO, ] which is choice (D). Choice (A) is incorrect because this is the zero-order rate law and the plot of [NO] is not linear. Choice (B) is incorrect because this is taken directly from the plot with 1/[NO:] on the y-axis. Choice (C) is incorrect because this is a first-order rate law and the plot of In[NO-] is not linear. Practice Questions Related to This: PQ-10 and PQ-11 127
Kinetics @ ACS SQ-6. ‘What changes when a catalyst is added to the reaction described by this energy diagram? (A) Tand II (B) Land 11 ©) Tonly (D) I only Knowledge Required: (1) Interpretation of reaction energy diagrams. (2) Meaning of catalyst and activation energy. Thinking it Through: Y ou are being asked to interpret a reaction energy diagram. Several parts of the diagram have been labeled. Examining the diagram, you recall what each labeled portion represents. 1 s the activation energy for the reaction in the forward direction. 11 is the activation energy for the reaction in the reverse direction. II1 is the difference in energy of the reactants and products, which is the AH for the reaction. The question asks you to determine which of these is affected by the addition of the catalyst. You remember that a catalyst provides an alternative pathway for the reaction. This alternative pathway has a lower activation energy. The presence of a catalyst does not affect the value of AH for the reaction. Because arrows I and I represent activation energies, these will be affected by the addition of a catalyst. The correct answer is choice (A). Choice (B) and (D) are incorrect because a catalyst does not affect the value of the enthalpy change, IIL Choice (C) is incorrect because a catalyst will affect the activation energy for both the forward and reverse reactions. Practice Questions Related to This: PQ-12, PQ-13, and PQ-14 SQ-7. Which statement regarding chemical reactions is true according to collision theory? (A) All molecular collisions result in chemical reactions. (B) Catalysts make individual collisions more effective. increasing reaction rates. (©) Proper orientation of molecules is required for collision to result in chemical reactions. D) Increasing the temperature of a reaction decreases the kinetic energy of molecules, making collisions more effective. | Knowledge Required: (1) Understanding of the collision theory of chemical reactions. (2) Knowledge of what factors influence the rate of chemical reactions. Thinking it Through: The question asks you to evaluate a series of statements about chemical reactions and collision theory. You decide to evaluate each option individually. You know that for a collision to lead to a chemical reaction it must satisfy two criteria. The collision must have sufficient energy for the reaction to occur. Also, the molecules must be in the proper orientation for the collision to be effective. Therefore, not all collisions result in a chemical reaction. Therefore choice (A) is incorrect. The presence of a catalyst does increase the rate of a reaction. However, the way a catalyst increases the rate of a reaction is not by making the collisions more effective. Instead a catalyst increases the rate of a reaction by providing an alternative route from reactants to products that has a smaller activation energy. Processes with smaller activation energies result in a faster reaction rate. Therefore choice (B) is incorrect. 128
Your preview ends here
Eager to read complete document? Join bartleby learn and gain access to the full version
  • Access to all documents
  • Unlimited textbook solutions
  • 24/7 expert homework help
cACS Kinetics —9Y Exams You know that one of the two requirements for a collision to lead to a chemical reaction is that the reactants must collide with the proper orientation. Therefore choice (C) is correct. Increasing the temperature often leads to an increase in the rate of a reaction. However, the reason is because as the temperature increases the average kinetic energy of the reactants increases. This can increase the rate of a reaction in two ways. First, there are more collisions that have the minimum energy needed for a reaction to occur. Second, the higher kinetic energy increases the number of collisions, which means the odds of a collision having the proper orientation increases. This increases the rate of a reaction. Therefore choice (D) is incorrect. Practice Questions Related to This: PQ-15, PQ-16, PQ-17, PQ-18, PQ-19, PQ-20, PQ-21, PQ-22, PQ-23, and PQ-24 SQ-8. The activation energy for a particular reaction is 83.1 kJ-mol™'. By what factor will the rate constant increase when the temperature is increased from 50.0 °C to 60.0 °C? (A) 253 (B) 1.00 © 0.927 (D) 0.395 Knowledge Required: (1) How to use the Arrhenius equation. (2) The relationship between activation energy and the value of the rate constant. Thinking it Through: You are being asked to determine the relative change in the value of the rate constant when the temperature changes. You recall the Arrhenius equation which relates the value of the activation energy, the temperature, and the rate constant. You also remember that all the temperature values must by in kelvin: k= A BT . . -E, 1 An alternative form of the equation is: Ink= (—)(—] +In(4) R T This form of the equation is useful if you have measured values of & at different temperatures. You recognize that this equation is in the form: y = mx + 5. If you plot In{k) versus 1/7 the slope of this plot is equal to (~<E./R). Another form of this equation, often called the two-point form, is used when you only have two values of k or temperature values. You get this form of the equation by subtracting the equation at the first temperature, T, from that at the second temperature, 75: Mz_.nk;(-_lg"-)[%}_(j-][%},nw_lnu, o) G You recognize that the ratio of k2/k) is what the problem is asking for and you substitute the given values into the equation. Note: you are careful to make sure the units of R and E, match and the temperature is in kelvin: wl ~83.141 - mol” (10001)( 1 1 ) k 83143 mol K" U 13 JA33315K 323.15K K, In| == |=0.927 k\ The correct answer is choice (A). Choice (B) is not correct because it is the answer if the units of £, are not converted to J. Choice (C) is not correct because it is the natural log (In) of the ratio of the rate constants. Choice (D) is not correct because it is the ratio ki/k. " =253 Practice Questions Related to This: PQ-25 and PQ-26 129
Kinetics .o ACS /WY Exams SQ-9. Consider the reaction 2NOs(g) + Falg) = 2NO:F(g) A proposed mechanism for the reaction is NO; + Fa= NO:F - F (slow) NO: + F= NO;F (fast) What is the rate law for this mechanism? A etk ®) KINO_TE] Tate =k ————— rate = LTI NOTIE] ¢ I (©) rate=kNO,][F] (D) rate=A[NO,J(F] Knowledge Required: (1) The fact that the slowest step in the reaction mechanism determines the rate law for the | reaction. (2) Being able to write a rate law expression from an elementary step. Thinking it Through: You are given a mechanism and are asked to write the rate law for the proposed mechanism. The slowest step in a reaction mechanism is the step with the highest activation energy and the species involved in this step determine the rate law for the reaction. You note that you are told the first step is the slowest. You know that you can write the rate law for this elementary step using the coefficients from the elementary step as the exponents in the rate law. You write the rate law for the slow step as: rate = ([NO, ][F, ] which is choice (C). Choice (A) is not correct because it is the cquilibrium expression for the reaction, not a rate law. Choice (B) is not correct because it uses the stoichiometry of the overali reaction. Choice (D) is not correct because it is not the derived from the slowest step. Practice Questions Related to This: PQ-27, PQ-28, and PQ-29 SQ-10. _ Consider this equilibrium: 280s(g) + Ox(g) = 280s(g) The forward reaction is proceeding at a certain rate at some temperature and pressure. When the pressure is increased. what might we expect for the forward reaction? (A) agreater rate of reaction and a greater yield of SO: at equilibrium (B) a greater rate of reaction and a same yield of SO; at equilibrium (©) a lesser rate of reaction and a smaller yield of SO; at equilibrium (D) alesser rate of reaction and a greater yield of SO; at equilibrium Knowledge Required: (1) How to use LeChatlier’s principle. (2) Qualitative understanding of rate laws. Thinking it Through: Y ou need to predict what will happen to an equilibrium system when it is subjected to a change. You know that a change in pressure may shift the equilibrium and depends on the moles of gaseous substances as reactants and products. An increase in pressure will shift the equilibrium in the direction that minimizes the increase in pressure. This is the side that has the fewest moles of gas. For this reaction, the side with fewer moles of gas is the product side. You realize that this means that an increase in pressure will cause the rate of the reaction producing SOs to increase. The correct answer is choice (A). Choices (B) and (C) is incorrect because SO; will increase. Choice (D) is incorrect because the rate of the reaction will increase. Practice Questions Related to This: PQ-30 130
cACS Kinetics Practice Questions (PQ) w‘f\“\ PQ-1. When the reaction: "Exp [CH:Cllo/M [H:000/M Initial Rate / M- CH;Cl(g) + H:0(g) CH:OH(g) + HCI(g) [ 0.100 0.100 0.182 was studied, the tabulated data were 2 0.200 0.200 1.45 obtained. Based on these data, what are the 3 0.200 0.400 5.81 reaction orders? (A) CH;Cl: first order H,O: first order (B) CH:Cl: first order H-0: second order (C) CH;CI: second order H»O: first order (D) CH:Cl: second order H>0: second order PQ-2. The reaction between acetone and bromine Ex [CH3COCHso [Br2Jo/ [H3O7o/ Initial Rate in acidic solution is represented by the P /M M M /M-s™! equation: 1 0.30 0.050 0.050 5.8%107 CH;COCH;(aq) + Bra(aq) + H:O"(aq) 2 0.30 0.100 0.050 5.8x107 products 3 0.30 0.050 0.100 1.2x104 The tabulated kinetic data were gathered. 4 0.40 0.050 0.200 3.2x107 Based on these data, the experimental rate law is (A) rate = {{CH;COCH;](Br2][H;0"] (B) rate = k[CH;COCH;][H;:O'] (C) rate = k[H;0°]* (D) rate = K[CH;COCH;][Br] PQ-3. Initial rate data for the reaction Exp [Hz]o/ M [ClzJo/M _Initial Rate / M-s™! 2Hx(g) + Cl(g) 2HCI(g) 1 0.0020 0.0050 2.5%107% are given in the table. What is the rate law | 2 0.0020 0.0025 1.3x107 for the reaction? 3 0.0015 0.0025 1.3x107 4 0.0050 0.0010 0.5%107 (A) rate = K[CLJ (B) rate =[Cly] (C) rate = k[Ha) (D) rate=k{H:)[Cl>] molecules. The rate law is rate =k[A2][B2] 1f the concentrations of both A and B> are doubled, the reaction rate will change by a factor of (A) % ®) 2. © 2. D) 4. W’,& PQ-4. The gas phase reaction, A + B, —2AB, proceeds by bimolecular collisions between Az and B> PQ-5. For the reaction of chorine and nitric oxide, 2NO(g) + Clx(g) 2NOCl(g) doubling the concentration of chlorine doubles the rate of reaction. Doubling the concentration of both reactants increases the rate of reaction by a factor of eight. The reaction is (A) first order in both NO and Clo. (B) first order on NO and second order in Cl,. (C) second order in NO and first order in Cl,. (D) second order in both NO and Cla. 131
Your preview ends here
Eager to read complete document? Join bartleby learn and gain access to the full version
  • Access to all documents
  • Unlimited textbook solutions
  • 24/7 expert homework help
Kinetics = ACS W Exams PQ-6. The decomposition of NOCI is second order. 2NOCI(g) 2NO(g) + Clx(g) The initial concentration of the reactant is 0.050 M. The rate constant equals 0.020 L-mol'-s™, what will the concentration be after 45 min? (A) 0.048M (B) 0.029M (©) 0.014M (D) 0.0054M PQ-7. Ifthe half-life of a reaction is independent of concentration, the reaction can be [ Ifirstorder | IIsecond order | I zeroorder | (A) TandIIonly (B) IIand Il only (C) Tonly (D) Ionly PQ-8. The activity of a radioisotope is 3000 counts per minute at one time and 2736 counts per minute 48 hours later. What is the half-life, in hours, of the radioisotope? (A) 831hr (B) 521hr (©) 361hr (D) 144hr PQ-9. After 55 years, what mass (in g) remains of a 200.0 g sample of a radioactive isotope with a half-life of 10.0 years? A) 022g (B) 44g ©) sig D) 170¢g PQ-10. If the reaction Os(g) Ox(g) + O(g) is first order in O3, which plot will be linear? (A) [Os] vs. time (B) 1/[0;] vs. time (€) [0:] vs. time (D) In[O5] vs. time PQ-11. The experimental data from a certain reaction gives these three graphs. What is the most likely order for this reaction? g z 5 time time time (A) zero (B) first (C) second (D) third PQ-12. Consider the reaction potential energy diagram. ‘What describes the catalyzed forward reaction pathway? Ea AH (A) 10kJ-mol™ ~15 kJ'mol™ (B) 10kJ-mol” 15 kJ-mol™! (€) 25kJ'mol”! —15 kJ-mol™! (D) 25kJ-mol”’ 15 kJ-mol"! 132
Your preview ends here
Eager to read complete document? Join bartleby learn and gain access to the full version
  • Access to all documents
  • Unlimited textbook solutions
  • 24/7 expert homework help
cACS Kinetics P Exams PQ-13. Which line segment represents the activation energy for the reaction between C and D to form A and B? T £ £ 5 = 5] Reaction Coordinate A) 1,2,&3 ®B) 2 ©) 2&3 D) 3 PQ-14. A certain reaction has AH = 75 ki-mol™' and an activation energy of 40 kJ'mol"'. A catalyst is found that lowers the activation energy of the forward reaction by 15 kJ-mol”. What is the activation energy of the reverse reaction in the presence of the same catalyst? (A) 25 kJ'mol”! (B) 60 kJ'mol"! (©) 90 kl'mol (D) 100 ki-mol™! PQ-15. Which statement best explains why the activation energy is changed by adding a catalyst? (A) The catalyst changes the reaction mechanism. (B) The catalyst changes the free energy of reaction. (C) The catalyst increases the kinetic energy of the reactants. (D) The catalyst lowers the reaction volume, increasing the concentration of reactants. M‘d PQ-16. Which reaction rate is more affected by a change in temperature? Reaction E./kJ-mol! [ Ha(g) + L(g) 2H(g) 173 I CH;CHO(g) CHa(g) + CO(g) 356 (A) reaction I because the activation energy is lower (B) reaction I because the number of moles of gas stays the same (C) reaction II because number of moles of gases increases as reaction goes forward (D) reaction II because the activation energy is higher PQ-17. What will increase the value of the rate constant for an elementary step? (A) adding a catalyst (B) raising the temperature (C) increasing the concentration of products (D) increasing the concentration of reactants 133
Your preview ends here
Eager to read complete document? Join bartleby learn and gain access to the full version
  • Access to all documents
  • Unlimited textbook solutions
  • 24/7 expert homework help
Kinetics we ACS M‘ PQ-18. (A) (B) ©) () MPQ—IQ. (A) (B) ©) D) ‘,«’vpd PQ-20. (A) (B) © (D) “P‘JPQ-ZI. [CY] (B) ©) (D) M PQ-22. “) ®) ©) (D) w“’"’d PQ-23. “) (B) ©) (D) Exams The Arrhenius equation describes the relationship between the rate constant, &, and the energy of activation, E, k= de” BT In this equation, 4 is an empirical constant, R is the ideal-gas constant, e is the base of natural logarithms, and 7 is the absolute temperature. According to the Arrhenius equation, at constant temperature, reactions with lower activation energies proceed more rapidly. at constant temperature, reactions with lower activation energies proceed less rapidly. at constant energy of activation, reactions at lower temperatures proceed more rapidly. at constant energy of activation, reactions with smaller values of 4 proceed more rapidly. ‘What will be the effect of increasing the temperature of reactants that are known to undergo an endothermic reaction? Both the rate of reaction and the value of the equilibrium constant increases. The rate of reaction increases and the value of the equilibrium constant decreases. The rate of reaction decreases and the value of the equilibrium constant increases. The rate of reaction increases and the value of the equilibrium constant is unchanged. The change in temperature from 10 °C to 20 °C is found to double the rate of a particular chemical reaction. How did the change in temperature affect the reaction? The number of molecules with sufficient energy to react increased. The number of molecules with sufficient energy to react decreased. The activation energy increased. The activation energy decreased. The value for the rate constant of a reaction can generally be expected to decrease with increasing temperature. increase with increasing temperature. decrease with increasing temperature only when the reaction is exothermic. increase with increasing temperature only when the reaction is exothermic. Two reactions with different activation energies have the same rate at room temperature. Which statement correctly describes the rates of these two reactions at the same, higher temperature? The reaction with the larger activation energy will be faster. The reaction with the smaller activation energy will be faster. The two reactions will continue to occur at the same rates. A prediction cannot be made without additional information. A catalyst increases the rate of a reaction by changing the mechanism of the reaction. increasing the activation energy of the reaction. increasing the concentration of one or more reactants. decreasing the difference in relative energy of the reactants and products. 134
Your preview ends here
Eager to read complete document? Join bartleby learn and gain access to the full version
  • Access to all documents
  • Unlimited textbook solutions
  • 24/7 expert homework help
Kinetics . ACS ¥ Exams M PQ-24. Which statement most accurately describes the behavior of a catalyst? (A) A catalyst increases AG of a reaction and hence the forward rate. (B) A catalyst reduces the AH of a reaction and hence the temperature needed to produce products. (C) A catalyst reduces the activation energy for a reaction and increases the rate of a reaction. (D) A catalyst increases the equilibrium constant and final product concentrations. PQ-25. The rate constants for a specific reaction at two different temperatures are given in the table. ‘What is the activation energy for the reaction? (A) -88.1 kJ'mol”’ (B) “127Kmol! (C) 127 Temperature _Rate constant 127 °C 3.0x10%s! 227°C 6.0x102s™" KJmol”! (D) 88.1kJmol”! PQ-26. Activation energy can be experimentally determined from the slope of the plot of (A) kversus I/T. (B) kversus In7. (C) Inkversus T. (D) Ink versus 1/T. M& PQ-27. Which energy diagram best matches the proposed mechanism for this exothermic reaction? E+S=ES (fast) ES—E+P (slow) (A) T (B) T 8 Reaction Progress —» © (D) Fnergy ~—> A Reaction Progress —» PQ-28. Consider the reaction Reaction Progress —» L Reaction Progress —» Clx(g) + HaS(aq) S(s) + 2H*(aq) + 2Cl(aq) The rate equation for this reaction is rate = k{CLy)[H.S} Which mechanisms is (or are) consistent with this rate equation (if any)? 1 Cly+ Hy8 H' +CI + CI* + HS® (slow) CI'+HS" H'+ Cl'+§ (fast) i HaS H™+ HS™ (fast) Cly+HS"—2CI+ H* +8 (slow) (A) Tonly (B) Monly (C) BothIand It (D) Neither Lor IT 135
Your preview ends here
Eager to read complete document? Join bartleby learn and gain access to the full version
  • Access to all documents
  • Unlimited textbook solutions
  • 24/7 expert homework help
Kinetics o ACS W Exams PQ-29. The decomposition of hydrogen peroxide in the presence of iodide ion is believed to occur via this mechanism. H:0:(aq) - I (aq) H:0(1) + 10(aq) H:05(aq) + 10-(ag) H:0(1) ~ Ox(g) + I(aq) In this mechanism, [ (aq) is (A) acatalyst. (B) areactant in the overall reaction. (C) the activated complex. (D) aproduct of the overall reaction. PQ-30. What is the relationship between the equilibrium constant (K.) of a reaction and the rate constants for the forward (k) and reverse (,) reactions in a single step reaction? (A) K=k B) Ko =klks © Ko =1liksh,) ® K=k-k 136
Your preview ends here
Eager to read complete document? Join bartleby learn and gain access to the full version
  • Access to all documents
  • Unlimited textbook solutions
  • 24/7 expert homework help
o ACS Kinetics Eak ol 2l o OFE Pownanpwnrs CwanOATREE® - flyEmms Answers to Study Questions RN wA»T - Ze >0 Answers to Practice Questions 11.B 21.B 12.A 22.A 13.D 23.A 14.D 24.C 15.A 25.D 16.D 26.D 17.B 27.C 18.A 28.A 19.A 29.A 20.A 30.B 137
Your preview ends here
Eager to read complete document? Join bartleby learn and gain access to the full version
  • Access to all documents
  • Unlimited textbook solutions
  • 24/7 expert homework help
Kinetics 138
Your preview ends here
Eager to read complete document? Join bartleby learn and gain access to the full version
  • Access to all documents
  • Unlimited textbook solutions
  • 24/7 expert homework help
Chapter 11 Equilibrium Chapter Summary: This chapter will focus on equilibrium including equilibrium constants and determining the amount of reactants and products at equilibrium. Specific topics covered in this chapter are: Definition of equilibrium Law of Mass Action K, and K. Equilibrium calculations Reaction quotient (Q) of a reaction e Le Chatlier’s Principle Previous material that is relevant to your understanding of questions in this chapter include: * Stoichiometry (Chapter 4) e Solutions and Aqueous Reactions, Part 1 (Chapter 5) * Solutions and Aqueous Reactions, Part 2 (Chapter 9) Where to find this in your textbook: The material in this chapter typically aligns to “Fundamentals of Equilibrium Concepts™ in your textbcok. The name of your chapter(s) may vary. Practice exam: There are practice exam questions aligned to the material in this chapter. Because there are a limited number of questions on the practice exam, a review of the breadth of the material in this chapter is advised in preparation for your exam. How this fits into the big picture. The material in this chapter aligns to the Big Idea of Equilibrium (8) as listed on page 12 of this study guide. Study Questions (SQ) SQ-1. What is the equilibrium expression for this reaction? Ni(CO),(g) = Ni(s) + 4CO(g) 4 . Ni(CO), Ni(CO), w g O8Nl o [Nico)] < ~[NiCo).] < [Ni(co),] [co]' ® 2 [Ni][co]* Knowledge Required: (1) How to determine an equilibrium expression from a balanced equation. Thinking it Through: The question asks you determine the equilibrium expression for a given reaction. The answer options are in Kc; therefore, you will consider the concentrations of the materials when constructing the equilibrium expression. An equilibrium constant is defined as the concentrations of products raised to a value equal to their stoichiometric coefficient in a balanced chemical equation divided by the concentrations of reactants raised to a value equal to their stoichiometric coefficients in a balanced chemical equation. Because the concentrations of solids and liquids do not change measurably, both are not included in the equilibrium expression. Therefore, when the concentration of gaseous or solution products are raised to their power you have a numerator of [COJ*; likewise for reactants, when the concentration of gaseous or solution reactants are raised to their power you have a denominator of [Ni(CO)s]. Combined, the numerator and denominator are Choice (A). Choice (B) is not correct because it includes a solid in the expression. Choice (C) is not correct because it has the reactants in the numerator and the products in the denominator, which is reversed. Choice (D) is not correct because it includes a solid in the expression and is the inverse of the balanced 139
Your preview ends here
Eager to read complete document? Join bartleby learn and gain access to the full version
  • Access to all documents
  • Unlimited textbook solutions
  • 24/7 expert homework help
Equilibrium _*ACS ¥ Exams chemical equation as written. Practice Questions Related to This: PQ-1, PQ-2, PQ-3, PQ-4, PQ-5, PQ-6, PQ-7, PQ-8, PQ-9, PQ-10, and PQ-11 SQ-2. Given the reaction and equilibrium constant: 2S0,(2) = 280,(g) + 0,(g) Kc=23x107 What is the equilibrium constant for this reaction at the same temperature? SO,(2) =280,(2)+ $0,(2) Kc=? (A) 12¢107 (B) 4.6x107 (©) 4.8x10* (D) 43x10° Knowledge Required: (1) The relationship between a balanced equation and an equilibrium constant. Thinking it Through: The question asks you to determine an equilibrium constant for a given balanced chemical equation given a constant for a related balanced chemical equation. You note that the chemical equation for which you are to determine an equilibrium constant is halved compared to the chemical equation for which the equilibrium constant is known. You can write the equilibrium expressions then for both equations and compare: [50.]'[0,] 250,(g) <= 250,(g) + O, g=l ] 1(g 1 (8) +0,(8) [so.f - 1 _[so0." v 50,(2) =50,(2) + 7 0(®) K'= [50.] =Kk =JK ‘When the reaction is halved, the equilibrium constant is raised to the 0.5 power (or the square root). Thus, the equilibrium constant needed is (2.3x10°7)** or 4.8x10°%, Choice (C). Choice (A) is not correct because it is the known equilibrium constant divided by 2. Choice (B) is not correct because it is the known equilibrium constant multiplied by 2. Choice (D) is not correct because it is the inverse of the known equilibrium constant. Practice Questions Related to This: PQ-12 $Q-3. Consider the equilibrium reactions: 250,(g) = 0,(2) = 280,(2) Ki 2CO(g) + 0,(g) =2C0,(g) K> What is the equilibrium constant, K, for this reaction? 250, (g) + 2CO, (g) = 280;(g) + 2CO(g) -( & p. - =K+ @ K_(KZJ ® K (K] © K=KxK, ® K=K+ Knowledge Required: (1) How to combine multiple chemical reactions and their respective equilibrium constants to form a new reaction and constant. Thinking it Through: The question asks you to determine the equilibrium constant for a chemical reaction that results from the combination of two reactions with given equilibrium constants. First, you note that in order to combine the given reactions, the second reaction must be reversed; therefore, the value of the equilibrium constant for the reverse reaction is the inverse of the given K or K5™'. 250,(g) + 0,(g) = 280,(g) K 2C0,(g) = 2CO(g) + 0, (8) K 280, (g) + 2CO, (g) = 280;(g) + 2CO(g) Kix K™ The desired reaction is now the sum of the first reaction and the inverse of the second reaction; when reactions are summed, the new equilibrium constant is the product of the two known equilibrium constants or K,* K;™! for the 140
Your preview ends here
Eager to read complete document? Join bartleby learn and gain access to the full version
  • Access to all documents
  • Unlimited textbook solutions
  • 24/7 expert homework help
s ACS Equilibrium reaction given, Choice (A). Choice (B) is not correct because it represents the inverse of the desired chemical reaction. Choice (C) is not correct because it does not account for the reverse of the second chemical reaction. Choice (D) is not correct because it is the sum of the two constants; however, when reactions are summed, the respective equilibrium constants are multiplied. Practice Questions Related to This: PQ-13 and PQ-14 SQ-4. Consider the reaction: X, (g) +2Y(g) = 2Z(g) . 12.00 moles of Z are placed in an evacuated 2.00- liter flask. After the reactants and products reach equilibrium, the flask contains 6.00 moles of Y. What is the equilibrium constant, K, for the reaction? (A) 0.333 (B) 0.667 (©) 150 (D) 3.00 Knowledge Required: (1) How to determine an equilibrium expression from a balanced equation. (2) How to calculate equilibrium amounts from an equilibrium expression. Thinking it Through: The question asks you to determine the equilibrium constant given a chemical reaction, the amount of the single species present before equilibrium, and the amount of one species after equilibrium is reached. First, you are told that there are 12.0 moles of Z initially, and 6.00 moles of Y at equilibrium. You then also consider that you know how the amounts will change because you begin with only product (so the change for Z will be negative and the change for X; or Y will be positive) and the amounts of these changes from the stoichiometry of the balanced equation (an increase of 2x because the coefficient for Y is 2, etc.). This is best summarized in a table: X2 + 2y = 2z Initial 0 0 12.00 Change +x +2x 2x Equilibrium +x 6.00 12.0 - 2x Now you can see that you have an equivalency of 2x = 6.00 or x = 3.00. Using this value for x, you can complete all values in the table: X2 + 2Y = 2Z Tnitial 0 0 12.00 Change +3.00 +2(3.00) —2(3.00) Equilibrium 3.00 6.00 6.00 You now have all of the molar amounts of the substances at equilibrium. To obtain the molar concentrations, you divide each value by 2.00 L (given in the problem): [Xa]=1.50M [Y]=3.00M (Z]=3.00M To determine the value of K, you write an equilibrium expression for this reaction using the concentration of the product raised to its coefficient divided by the concentrations of the reactants raised to their coefficients; you can then substitute the equilibrium concentrations and calculate K: oz oMy 1 DCIYE (1,50 M)(3.00 MY 1.50 Choice (A) s not correct because it is determined using moles and not molar concentrations. Choice (C) is not correct because it is determined using the inverse of K. Choice (D) is not correct because it is determined using moles and the inverse of K. =0.667 Choice (B) Practice Questions Related to This: PQ-15 and PQ-16 SQ-5. Phosgene decomposes into carbon monoxide and elemental chlorine. If the initial concentration of COCly(g) is 0.50 M, what is the equilibrium concentration of CO(g)? COCl,(g) = CO(g) +Cly(g) Ke=6.6x10* (A)_1.8x10%M (B) 9.1x10°M 17x10%M (D) 6.6x10°M 141
Your preview ends here
Eager to read complete document? Join bartleby learn and gain access to the full version
  • Access to all documents
  • Unlimited textbook solutions
  • 24/7 expert homework help
Equilibrium w. ACS Exams Knowledge Required: (1) How to write an equilibrium expression from a balanced equation. (2) How to use the equilibrium expression to find equilibrium amounts. Thinking it Through: The question asks you to determine the equilibrium concentration of a product given that only the reactant is present before it is allowed to reach equilibrium. You can assume that the reaction is done at constant volume and therefore concentrations are proportional to amount. Begin by setting up a table of initial-change-equilibrium concentrations. Initial concentrations are provided in the question. Mole ratios are used to determine the changes in concentration. Equilibrium values are determined by the addition/subtraction of the initial and change values: coc, = co Cly Tnitial 0.50 M 0 0 Change X +x X Equilibrium 0.50 —x x x The equilibrium expression for this reaction is the concentration of products raised to their respective coefficients divided by the concentration of the starting material raised to its respective coefficient: « - 1C0ICL] < [cocL] You then substitute the values obtained into this expression: 6.6x10™ = %)— .30 -x 2 And simplified further to: 6.6x10™ =—* 0.50-x At this point, you may be inclined to continue the algebra to set up the equation to use the quadratic equation. However, you also know you may be able to simplify the process. The value of the equilibrium constant is very small. You know this means that most of the species at equilibrium are reactants with only a very small quantity of products if you began with only reactants (which you did). Therefore, you then note that any change to the initial concentration of COCI; will be very small, leaving the initial concentration (0.50 M) essentially unchanged or: COCl, = co Ch Tnitial 0.50 M 0 0 Change —x +x +x Equilibrium 0.50 - x~0.50 x x ‘Which then makes the equilibrium expression: 6.6x10° = w ~ M 0.50-x 050 Which can now be solved without the quadratic equation: 6.6x10" = (L 050 0.50 Of course, you also know there is nothing wrong with continuing from before and setting up the equation and using the quadric equation: 6.6x10°(0.50~x) = x* *+6.6x107 (x)-3.3x10" The quadratic equation can then be used to determine possible values of x: £1.8x107. concentrations of the products cannot be negative, x must be equal to +1.8x 10~ and thus the final concentration of CO is 1.8x10 M, choice (A), which was the same answer you obtained using the approximation. Note: You can check your approximation by using the value of x you calculated and using this in the equilibrium expression to calculate K. the value will be the same as given if the approximation can be used. You can also approximate in the reverse if the value of K. is very large and you only begin with product (so the change to the product concentration will be small). Choice (B) is not correct because it would be the answer if *“(2x)*” was the numerator of K. Choice (C) is not correct because it would be the answer if “2x” was the numerator of K. Choice (D) is not correct because the final concentration of CO is not the value of K. Practice Questions Related to This: PQ-17, PQ-18, and PQ-19 x=1.8x10" 142
Your preview ends here
Eager to read complete document? Join bartleby learn and gain access to the full version
  • Access to all documents
  • Unlimited textbook solutions
  • 24/7 expert homework help
ACS Equilibrium Exams SQ-6. BrCl(g) is in equilibrium with Brx(g) and Cla(g) at 25.0 °C: 2BrCl(g) = Br,(g) + Cl,(8) K, =0.130 Initially, in a closed container at 25.0 °C, BrCl(g) has a partial pressure of 0.400 atm and Brx(g) and Cly(g) each have partial pressures of 0.800 atm. What is the partial pressure of BrC(g) once the system reaches equilibrium? (A) 0419 atm (B) 0.781 atm (C) 1.16atm (D) 121 atm Knowledge Required: (1) How to determine K;, from a balanced equation. (2) How to calculate equilibrium partial pressures from a K;, expression. Thinking it Through: The question asks you to determine the partial pressure of a species given initial partial pressures of the starting material and products. Because this is a closed system, partial pressures are proportional to amount and therefore changes in partial pressures are proportional to changes in moles of a given species. To decide which way the reaction proceeds, you calculate a value of Q using the initial pressures given in the question: P, 0= B dolF )y Y o —(0'800)(0'2300) =16 (K, =0.130) Bl (0.400) Because Q is greater than K, the reaction will go to the left. You then continue by setting up a table of initial-change-equilibrium (ICE) partial pressures. Initial partial pressures are provided in the question. Your decision earlier that the reaction will proceed to the left is indicated in the ICE table by the negative (-) signs in the change line for the products and the positive (+) sign for the reactant. Mole ratios are used to determine the changes in partial pressures (so +2x for 2BrCl and —1x for 1Brs, etc.). Equilibrium values are determined by the addition/subtraction of the initial and change values. 2BrCl(g) = Bn(g) Ch(g) Initial 0.400 atm 0.800atm 0.800 atm Change +2x -x -x Equilibrium 0.400 +2x 0.800—x 0.800 - x The Kj expression for this reaction is the partial pressures of products raised to their respective coefficients divided by the partial pressure of the reactant raised to its respective coefficient: You then substitute the values obtained into this expression: .800—x)(0.800— 0.130 = (800-3)(0800-x) (0.400+ 2x) The expression is then further simplified: (The perfect square is used in the simplification.) 0.800-x)" —x Y 0.130= A0800=3)" m;(,:[%] (0.400+2x) 0.400+2x 0.800 V0130 = | 0.361(0.400 +2x) = 0.800 (0A400+2x ( ) * 0.144+0.722x = 0.800-x 1.722x = 0.656 x=0381 Replacing this value of “x” into the table, we determine the partial pressures at equilibrium; 2BrCl(g) = Bra(2) Cly(g) Initial 0.400 atm 0.800 atm 0.800 atm Change +2(0.381 atm) (0.381atm) (0381 atm) Equilibrium 1.16 atm 0.419 atm 0.419 atm Thus, the equilibrium partial pressure of BrCI(g) is 1.16 atm, Choice (C). Choice (A) is not correct because it is the partial pressure of either product. 143
Your preview ends here
Eager to read complete document? Join bartleby learn and gain access to the full version
  • Access to all documents
  • Unlimited textbook solutions
  • 24/7 expert homework help
oo Equilibrium . ACS % Exams Choice (B) is not correct because the answer comes from adding one-half of x to the initial partial pressure of BrCl(g) instead of a full x; this is likely confused with subtracting one-half of x from either product partial pressure to determine their equilibrium partial pressures. Choice (D) is not correct because K, did not include the “2” exponent for the partial pressure of the reactant. Practice Questions Related to This: PQ-20 and PQ-21 SQ-7. ‘Which will drive the equilibrium to form more Cu(s)? Cu,O(s) + CO(g) =2Cu(s) + CO, (2) (A) remove CO(g) (B) remove CO(g) (C) add a catalyst (D) increase the volume of the container Knowledge Required: (1) Le Chatelier’s principle. Thinking it Through: The question asks you to determine what would influence the equilibrium such that more product is formed. Le Chatelier’s principle would suggest that removing a reactant would lead to the formation of more reactants; therefore, choice (A) is incorrect. Adding a catalyst, choice (C), would result in the rate of the reaction changing without impacting the position of equilibrium. Le Chatelier’s principle would suggest that increasing the volume would favor the side of the reaction with more moles of gas; however, there is the same number of moles of gas for both the reactants and products of this reaction, and choice (D), therefore, is incorrect. Finally, Le Chatelier’s principle would suggest that removal of a product would lead to the formation of more product; therefore, choice (B) is correct, i.e., “remove COx(g)” would lead to the formation of more Cu(s). Practice Questions Related to This: PQ-22, PQ-23, PQ-24, PQ-25, PQ-26, PQ-27, PQ-28, PQ-29, and PQ-30 Practice Questions (PQ) a PQ-1. When the reversible reaction Na(g) + Ox(g) == 2NO(g) has reached a state of equilibrium, (A) no further reactions occurs. (B) the total moles of products must equal the remaining moles of reactant. (C) the addition of a catalyst will cause formation of more NO. (D) the concentration of each substance in the system will be constant. PQ-2. Xenon tetrafluoride, XeFs, can be prepared by heating Xe and F together: Xe(g) + 2, (g) = XeF, () What is the equilibrium constant expression for this reaction? A) g - XE] ® g - XE] (o g o [XE] () o _[Xe]E] © [XellR,] ©2AXe]lR,] © [Xe]RT ¢ [XeR] M\ PQ-3. What is a proper description of chemical equilibrium? (A) The reaction has stopped. (B) The concentrations of products and reactants are equal. (C) The rates of the forward and reverse reactions are zero. (D) The rates of the forward and reverse reactions are the same. 144
Your preview ends here
Eager to read complete document? Join bartleby learn and gain access to the full version
  • Access to all documents
  • Unlimited textbook solutions
  • 24/7 expert homework help
- ACS Equilibrium ¥ Exams PQ-4. What is the K. for this reaction? P,(s) + 6Cl,(g) = 4PCl,(g) @ g oL ® g - PN o g _[PCL ® g -[RJCL] ©CL) <RI CeLy < e PQ-5. What is the K, for this reaction? 280, (g) = 250,(g) + O,(g) W _Rah ® © KD=(2PM,) A, TR, o, (2R, PQ-6. What is the equilibrium expression for this reaction? 2C(s) + 0,(g) = 2CO(g) @ g =19 ®) g _1COF g -_[2€0) ®) g [Cor ©[Q0,] ©[CT[0,] © [20)0,] © 0] M PQ-7. The value of an equilibrium constant can be used to predict each of these except the (A) extent of a reaction. (B) direction of a reaction. (C) time required to reach equilibrium. (D) quantity of reactant(s) remaining at equilibrium. PQ-8. Chemical equilibrium is the result of (A) astoppage of further reaction. (B) the unavailability of one of the reactants. (C) opposing reactions attaining equal speeds. (D) formation of products equal in mass to the mass of the reactants. PQ-9. The photosynthetic conversion of CO> and O, can be represented by the reaction shown. What is the equilibrium expression for this reaction? 6CO, (g) + 6H,0(1) = C,H,,0,(s) + 60, () @) g -[CO:TICH.O0] ® g _[CO.1 © [0,T[H,0F oy © g . L0:JH.0F ) g 10T © [CO,JICH,,04) ©co,r” PQ-10. Which statement best describes general equilibrium? (A) Equilibrium is reached when the reaction stops. (B) There is only one set of equilibrium concentrations that equals the K. value. (C) At equilibrium, the rate of the forward reaction is the same as the rate of the reverse reaction. (D) At equilibrium, the total concentration of products equals the total concentration of reactants. 145
Your preview ends here
Eager to read complete document? Join bartleby learn and gain access to the full version
  • Access to all documents
  • Unlimited textbook solutions
  • 24/7 expert homework help
Equilibrium 2 ACS ? Exams PQ-11. For this chemical reaction at 25 °C, the concentration of the product is the concentration of f reactants at equilibrium. C,H,(g) +H,(&) = C,H(2) Kc=13x10" (A) less than (B) equal to (C) greater than (D) is not able to be compared to PQ-12. In which case does K. = K,? (A) 2A(g) + B(s)=2C(s) + D(g) (B) 2A(g) + B(s) = C(s) +2D(g) (C) 3A(g) +B(s) =22C(s) +2D(g) (D) K. and K, are equivalent in more than one of the above PQ-13. Given the equilibrium constants for these reactions: a,#“d 4Cu(s) + 0,(8) = 2Cu,06) K, 2Cu0(s) = Cu,0(5) + 30,(8) K, What is the value of K. for this reaction? 2Cu(s) + O, (g) = 2CuO(s) K. K, @ K ® © & <., ® g ok, K, VK PQ-14. Given the equilibrium constants for these reactions: 2CH, (2) = C,H,(g) + H,(2) CH,(g) + H,0(g) = CH,;OH(g) + H,(g) K. What is the value of K. for this reaction? 2CH,OH(g) + H, (g) = C,H, () + 2H,0(g) (A) 9.5x10"7 (B) 2.9x107° (C) 3.4x10° (D) 1.2x10* 9.5x107"3 2.8x102 PQ-15. Carbon monoxide gas reacts with hydrogen gas at elevated temperatures to form methanol. CO(g) + 2H, (g) == CH,OH(g) ‘When 0.40 mol of CO and 0.30 mol of H; are allowed to reach equilibrium in a 1.0 L container, 0.060 mol of CH;OH are formed. What is the value of K.? (A) 0.50 (B) 098 © 17 D) 54 PQ-16. At equilibrium in a 10 L vessel, there are 7.60x10% moles of SO, 8.60x10~ moles of O, and 8.20x107 moles of SO3. What is the equilibrium constant K. under these conditions? 250, (g) + O,(2) = 280,(2) (A) 125 ®) 135 125 ) 135 146
Your preview ends here
Eager to read complete document? Join bartleby learn and gain access to the full version
  • Access to all documents
  • Unlimited textbook solutions
  • 24/7 expert homework help
> ACS Equilibrium Exams PQ-17. A mixture of 2.0 mol of CO(g) and 2.0 mol of HO(g) was allowed to come to equilibrium in a 10.0-L. flask at a high temperature. If K. = 4.0, what is the molar concentration of Hx(g) in the equilibrium mixture? CO(g) + H,0(g) = CO,(g) + H,(g) (A) 0.67M (B) 0.40M (©) 020M (D) 0.13M PQ-18. 1fa 1.0 L flask is filled with 0.22 mol of N; and 0.22 mol of O, at 2000°C, what is [NO] after the reaction establishes equilibrium? (K. = 0.10 at 2000°C) N, () + 0,() = 2NO(g) (A) 0.034M (B) 0.060 M (©) 0.079M (D) 0.12M PQ-19. For the reaction, A(g) =B(g) + C(g), 5 moles of A are allowed to come to equilibrium in a closed rigid container. At equilibrium, how much of A and B are present if 2 moles of C are formed? (A) 0 moles of A and 3 moles of B (B) 1 mole of A and 2 moles of B (C) 2 moles of A and 2 moles of B (D) 3 moles of A and 2 moles of B M PQ-20. The equilibrium constant, K. = 3.8x10°* for the reaction, L(g)=2(g) What is the state of the system if [[»(g)] = 1.0 M and [I(g)} = 1.0x10~* M? (A) The system is at equilibrium. (B) The system is shifting towards products. (C) The system is shifting towards reactants. (D) There is insufficient information to describe the status of the system. M PQ-21. For the reaction: LF,(g) = F(g), a reaction mixture initially contains equal amounts of Fx(g) and F(g) in their standard states. If K, = 7.55x1072 at this temperature, which statement is true? A) Q <K, and the reaction proceeds towards the reactants. ®) 2<K, and the reaction proceeds towards the products. (©) Q =K, and the reaction is at equilibrium. D) Q> K, and the reaction proceeds towards the reactants. PQ-22. Consider this reaction at equilibrium. w“"" 250,(g) + 0,(2) = 250,(2) AH=-198KJ ‘Which change would cause an increase in the SO; / SO mole ratio? (A) adding a catalyst (B) removing Ox(g) (C) decreasing the temperature (D) decreasing the pressure 147
Your preview ends here
Eager to read complete document? Join bartleby learn and gain access to the full version
  • Access to all documents
  • Unlimited textbook solutions
  • 24/7 expert homework help
Equilibrium ACS Exams PQ-23. At298 K, the equilibrium constant for this reaction AGe (kJ-mol™") H,(2) + $0,(g) = H,0()) H;0(l) -237 H:0(g) -229 (A) has a value of 1.0 at equilibrium. (B) is larger than the Kq for H,(g) + +0,(g) = H,0(2) . (C) cannot be computed since data on O, and H: are not provided. (D) will have the same value as the Keq for H,(g) + +0,(g) = H,0(g) M“@ PQ-24. Which factors will affect both the position of equilibrium and the value of the equilibrium constant for this reaction? N,(g) +3H,(g) = 2NH,(g) AH=-92kJ (A) increasing the volume of the container (B) adding more nitrogen gas (C) removed ammonia gas (D) lowering the temperature PQ-25. Consider this reaction carried out at constant volume. 250,(8) + O.(e) = 250,(5) AH=-198KJ The concentration of Ox(g) at equilibrium increases if (A) SO is added to the system. (B) SO is added to the system. (C) the temperature of the system is lowered. (D) aninert gas is added to the system. M@ PQ-26. Oxygen and ozone are allowed to come to equilibrium in the exozhermic reaction 20,(2) =30,(g) ‘Which change will increase the numerical value of the equilibrium constant, K? (A) adding more Ox(g) (B) adding a suitable catalyst (C) decreasing the temperature (D} increasing the volume of the container A PQ-27. This reaction reaches equilibrium in a closed container. What happens if the volume of the container vfl"’»’ is decreased at constant temperature? CH,(g) = H,0(g) = COg) + 2H.(g) (A) No change occurs. (B) The equilibrium constant increases. (C) The reaction shifts towards products. (D) The reaction shifts towards reactants. M PQ-28. For this equilibrium, what changes will favor formation of products? Ny(@)+3H,(9) = 2NH,(g) AH=-92k) (A) decrease temperature, increase pressure (B) decrease temperature, decrease pressure (C) increase temperature, increase pressure (D) increase temperature, decrease pressure A PQ-29. The table to the right gives the relative concentration of reactants X + 7 ¥ Xz 0.#”" and products at equilibrium for the generic reactionat two . (300K 07 03 08 temperatures. The equilibrium constant is 500K 1.0 0.6 0.5 (A) negative at 500 K. (B) larger at 500 K than at 300 K. (C) smaller at 500 K than at 300 K. (D) the same at 500 K and at 300 K. 148
Your preview ends here
Eager to read complete document? Join bartleby learn and gain access to the full version
  • Access to all documents
  • Unlimited textbook solutions
  • 24/7 expert homework help
:ACS Equilibrium % Exams A PQ-30. A catalyst is added to a system at equilibrium. Which statement is TRUE? M“ (A) The temperature will decrease. (B) The equilibrium constant will increase. (C) The concentration of products will decrease. (D) If the system is disturbed, it will return to equilibrium faster. 149
Your preview ends here
Eager to read complete document? Join bartleby learn and gain access to the full version
  • Access to all documents
  • Unlimited textbook solutions
  • 24/7 expert homework help
Equilibrium cACS Exams Answers to Study Questions 1. A 5. A 2.C 6. C 3. A 7. B 4. B Answers to Practice Questions 1. D 11.C 2.C 12.B 3. D 13.A 4.C 14.D 5. A 15.D 6. D 16.D 7.C 17.D 8. C 18.B 9.D 19.D 0. C 20.B 150 21.D 22.C 23.B 24.D 25.B 26.C 27.D 28.A 29.C 30.D
Your preview ends here
Eager to read complete document? Join bartleby learn and gain access to the full version
  • Access to all documents
  • Unlimited textbook solutions
  • 24/7 expert homework help
Chapter 12 Acids and Bases Chapter Summary: This chapter will focus on acids and bases including acid-base models, determining an acid dissociation constant, and calculating pH and pOH for a given acid dissociation. Specific topics covered in this chapter are: e Acid-base models (Arrhenius, Bronsted-Lowry, Lewis) * pHand pOH « Conjugate acid-base pairs * Titrations and neutralizations e Buffers Previous material that is relevant to your understanding of questions in this chapter include: * Solutions and Aqueous Reactions, Part 1 (Chapter 5) * Solutions and Aqueous Reactions, Part 2 (Chapter 9) * Equilibrium (Chapter 11) Common representations used in questions related to this material: Name Example Used in questions related to Titration curves N titrations and buffers n 10 z 8 s 4 2 o 3 20 a0 6 volume NaOH added (mL) Where to find this in your textbook: The material in this chapter typically aligns to “Acid-Base Equilibria” in your textbook. The name of your chapter(s) may vary. Practice exam: There are practice exam questions aligned to the material in this chapter. Because there are a limited number of questions on the practice exam, a review of the breadth of the material in this chapter is advised in preparation for your exam. How this fits into the big picture: The material in this chapter aligns to the Big Idea of Equilibrium (8) as listed on page 12 of this study guide. Study Questions (SQ) SQ-1. Which set represents a conjugate acid/base pair? (A) HCIO/HCIO, (B) H:PO,/HPO (C) H;O"/OH- (D) NCI/NCl, Knowledge Required: (1) Definition of conjugate acid/base pairs from the Bronsted-Lowry acid-base model. Thinking it Through: You are asked in the question to evaluate each answer option to determine which set is a conjugate acid/base pair. Recall from the Bronsted-Lowry acid-base model that conjugate acid/base pairs differ in 151
Your preview ends here
Eager to read complete document? Join bartleby learn and gain access to the full version
  • Access to all documents
  • Unlimited textbook solutions
  • 24/7 expert homework help
Acids and Bases . ACS W Exams one H". For choice (A), the two species differ by an oxygen atom; these are a set of analogous acids. For choice (B), the two species differ by an H™; therefore, choice (B) is correct. For choice (C), the two species differ by two H™ ions: while these species are related by H™, the difference in H is one too many. For Choice (D), the two species differ by a chlorine atom. You can also conceptualize this question using the reaction of each species in water to find the conjugate base (and this is good practice): Choice (A): HCIO(aq)+H,0(1) = CIO™ (aq)+H,0" (aq) acid conjugate base (not HCIO: as given in the problem) Choice (B): H.PO, (aq) +H,0(1) = HPO}" (aq) + H,0" (aq) acid conjugate base Choice (C): H,0" (aq)+ H,0(1) = H,0(1)+ H,0" (aq) acid conjugate base (nor OH- as given in the problem) Choice (D): NCI; would not behave as a Bronsted-Lowry acid Practice Questions Related to This: PQ-1, PQ-2, PQ-3, PQ-4, PQ-5, PQ-6, PQ-7, and PQ-8 5Q-2. What is the pH of a 0.820 M aqueous NH; solution? Ky, (NHs) = 1.8x10 (A) 242 (B) 925 (©) 1158 ®) 1391 Knowledge Required: (1) How to write a balanced equation for an aqueous acid-base reaction. (2) How to write the base dissociation constant expression (K4) for a balanced chemical equation. (3) How to use the K, expression to determine equilibrium concentrations. (4) How to convert between pH and pOH. Thinking it Through: You are asked to determine the pH of a given aqueous solution for a known base and a known K. This problem has several steps: First, you write a balanced equation for the acid-base reaction; recall that you are told that the reaction is in water, and thus water will act as an acid with the given base: NH, (aq) + H,0(1) = NH; (aq) + OH'(aq) Next, you use the balanced acid-base equation to determine the Ky, expression: concentrations of products raised to power of their stoichiometric coefTicients divided by the concentrations of reactants raised to power of their stoichiometric coefficients. Note: pure liquids and solids are not included in the K, expression (omitting water). NH]J[OH" PR [NH,] Next, you construct an initial-change-cquilibrium (ICE) table. Initial concentrations are provided in the question. Stoichiometric cocfficients from the balanced chemical equation are used to determine the changes in concentrations. Equilibrium values are determined by the addition/subtraction of the initial and change values. NHjaq) = NHi(ag) + OH-(aq) Initial 0.820 0 0 Change —x +x +x Equilibrium 0.820 - x x x Becausc acid and base ionization constants are only about 95% accurate, you can compare the ratio of the equilibrium constant to the initial concentration of the acid or base. If the ratio is less than 2.5%x1073, you can . K. 3 te or: - 2.5x10 approximate or: facid], <2.5x% oase], K,(NH;) 1.8x10" [NH,], 0820M Therefore, you can assume that x will be sufficiently small compared to the initial concentration of NHj; therefore, the change to [NH;]y is small and the approximation is [NH3Jo = [NH:]equitibrivm 01 0.820—x ~0.820 . For this question: =2.2x10"" which is less than 2.5x10* (so you can approximate) You now substitute the equilibrium concentrations into the K, expression: 152
Your preview ends here
Eager to read complete document? Join bartleby learn and gain access to the full version
  • Access to all documents
  • Unlimited textbook solutions
  • 24/7 expert homework help
o ACS Acids and Bases 18x10° =% 0.820 The expression is then further simplified and solved: 15x107 =x* x=%384x10" Because x is the final concentration of each of the products and because a concentration cannot be negative, then x is equal to +3.84x10* M which is [NH:"Lequioriun and [OH Jequiitriom. Next, you determine the pOH from the concentration of [OH]: POH = —log([OH"]) = ~log(3.84x10 ") = 2.42 Finally, you convert from pOH to pH knowing that the sum of pH and pOH is equal to 14 for an aqueous solution. pOH +pH = 14 pH = 14— pOH pH=14-242=11.58 Which is choice (C) Choice (A) is not correct because it is the pOH of the solution. Choice (B) is not correct because it was determined using the K, value as the concentration of OH™ when determining pOH. Choice (D) is not correct because it was determined using the initial concentration of NH; as the concentration of OH~ when determining pOH. Practice Questions Related to This: PQ-9, PQ-10, PQ-11, PQ-12, and PQ-13 SQ-3. ‘Which acid is the weakest in aqueous solution? (A) acetic acid (K, = 1.8x10°%) (B) formic acid (K, = 1.8x10%) (C) hydrocyanic acid (K; = 6.2x10 ') (D) nitrous acid (K, = 4.5x107) Knowledge Required: (1) How to write an acid dissociation equilibrium expression. (2) Relationship between acid dissociation constants (K) values and acid strength. Thinking it Through: You arc asked to evaluate the given acids and determine which acid is the weakest. You recall that K is expressed as the concentration of product(s) raised to the power of their stoichiometric coefficients (from the balanced chemical equation) divided by the concentration of reactant(s) raised to power of their stoichiometric coefficients. Thus, larger values of K, are associated with a larger product to reactant ratio. Conversely, smaller values of K, are associated with a smaller product to reactant ratio. For all of the choices, the values of K, are less than one (so for all of these, starting with only reactants, at equilibrium, [reactants] > [products]). However, you know you can still determine relative acid strength when all K, values are less than one by determining which value is the largest and how that corresponds to acid strength. Generically: A” || H,0" HA (aq)+H,0(1) = A™(aq) +H,0" (aq) K, :% The larger the value of K, , the highér [HSO‘], the stronger the acid Thus, small values of K, are associated with weak acids with the smallest K, value associated with the weakest acid. You consider the answer options and note that hydrocyanic acid has the smallest K, (6.2x10 %) and conclude that it is the weakest of all the acids given as answer options; therefore, choice (C) is correct. Practice Questions Related to This: PQ-14 and PQ-15 SQ-4. Which anion is the most basic? (A) Clo0” (B) CIO; (€) Cios (D) ClOs Knowledge Required: (1) How to determine acidity and basicity from an analogous series of oxoacids. Thinking it Through: You are asked in the question to evaluate a series of bases to determine which is the most basic. The bases are analogous, considering the answer options from (A) to (D), each has one more oxygen than the previous; thus, this a series of oxoacids. The general trend for a series of oxoacids is that as the number of 153
Your preview ends here
Eager to read complete document? Join bartleby learn and gain access to the full version
  • Access to all documents
  • Unlimited textbook solutions
  • 24/7 expert homework help
Acids and Bases =" ACS @ Exams oxygens increases (or more oxygen atoms in the acid) the acid gets stronger. This is due to the increase in electron withdrawing power of the added oxygens, which causes the O-H bond to weaken and the acid to get stronger. Increasing number of oxygen atoms weakest Increasing electron withdrawing ————> strongest acid : G—g—H o=G—0—H Weakening of O-H bond Increasing acid strength You can use the relative strength of the acid to determine the relative strengths of the conjugate bases as you know the value of K, for the conjugate base is related to the K, value of the acid through: Kix Ky =K. Because K., is a constant, at a given temperature, the larger the K, value (stronger the acid) the smaller the K value (weaker the conjugate base). Therefore, the conjugate base of the weakest acid (fewest oxygen atoms) will be the strongest base choice (A). The next strongest base would be the conjugate base of the next weakest acid choice (B). The pattern would continue. the ranking of the base strength is: strongest base weakest base CIO~> CIO; > CIO;~ > ClO4~ Although not part of this question, when considering the structure of acids and relative strength, you can also be provided with the same number of oxygen atoms, but differing halogens, for example comparing HC1O4 and HBrO. Here, the difference in acid strength is due to the higher electronegativity of chlorine compared to bromine with HCIOj as the stronger acid. Practice Questions Related to This: PQ-16 and PQ-17 SQ-5. What is Ky of F? (K. of HF is 6.8x107%) (A) 6.8x10"° (B) 1.5x10° (C) 6.8x10% D) 1.5%x10°" Knowledge Required: (1) Relationship between K, and K for a conjugate acid-base pair. Thinking it Through: You are asked in the question to determine the Ky, for a species given a known K, for the conjugate acid of the species. To solve this problem, it is necessary to recall the relationship between K. and K} for an acid and its conjugate base dissociated in water: HF(aq) <= H' (aq) + F (aq) X, F"(aq)+H,0(1) == HF(aq) +OH (aq) K, H,0(1) = H* (aq)+OH" (aq) K, =K, xK, " Therefore: K, = ;- - E%i_?o‘* ~1.5x107" which is choice (D). Choice (A) is not correct because it is determined by dividing K, by K., which is the inverse of K. Choice (B) is not correct because it is determined by taking the inverse of K. Choice (C) is not correct because it is the value of K,. Practice Questions Related to This: PQ-18 SQ-6. ‘Which substance will dissolve in water to produce an acidic solution? ‘p‘af" (A) FeCly (B) Na;0 (C) NaC:Hs0, (D) NH; Knowledge Required: (1) Dissociation of ions in aqueous solution. (2) Relative acidity/basicity of common species and ions. 154
Your preview ends here
Eager to read complete document? Join bartleby learn and gain access to the full version
  • Access to all documents
  • Unlimited textbook solutions
  • 24/7 expert homework help
= ACS Acids and Bases @ Exams Thinking it Through: You are asked in the question to evaluate a series of compounds to determine which will produce an acidic solution upon dissolution in water. The method you will use to do this is (in turn for each compound): 1. Consider the ions formed when the compound dissolves in water 2. Consider the reaction of each ion with water (hydrolysis) 3. Consider the acidic/basic ions Choice (A) - FeCl3 Step 1 FeCly(s) > Fe¥*(aq) + 3Cl(aq) Step2 Fe¥(aq) + 6H,0(1) = Fe(H:0)s(aq) = Fe(H:0):0H (aq) + H'(aq) Cl(aq) + H2O(1) = no reaction (HCl is a strong acid) Step 3 Fe'" is an acidic cation and CI" is a neutral anion FeCl; produces an acidic solution Choice (B) — Na,O Step 1 Na;O(s) > 2Na*(aq) + O*(aq) Step2 Na*(aq) + HyO() = no reaction (NaOH is a strong base) 0%(aq) + H0(l) = 20H~ Step 3 Na” is neutral cation and O is a basic anion Na;O produces a basic solution Choice (C) NaCH:0» Step 1 NaC;H:0x(s) = Na*(aq) + C;H305 (aq) Step2 Na(aq) + H;O(l) = no reaction (NaOH is a strong base) C2H;0:7(aq) + HO(I) = HC;H;0x(aq) + OH(aq) Step 3 Na” is neutral cation and C,H3O;" is a basic anion NaC>H;0; produces a basic solution Choice (D) NH; is a molecular compound and not a salt. Instead for this compound, you will consider the reaction of NHjs is water directly (omitting the dissociation into ions): NHs(aq) + H20(aq) = NHs'(aq) + OH(aq) which produces a basic solution. Choices (B), (C), and (D) all lead to the formation of hydroxide anions and basic solutions. Therefore, Choice (A) is correct because it leads to the formation of hydronium ion (H;0"). Practice Questions Related to This: PQ-19, and PQ-20 SQ-7. An acetate buffer contains equal volumes of 0.35 M HC;H;0; (pKa = 4.74) and 0.55 M NaC,H:0,. What is the pH of the buffer? (A) 454 (B) 4.74 (©) 494 () 7.00 Knowledge Required: (1) How to use the Henderson-Hasselbalch equation. (2) How a buffer works. Thinking it Through: You are asked in the question to determine the pH of a buffer solution given the pK, of the acid, concentrations of the acid and its conjugate base. The Henderson-Hasselbalch equation is used to relate pH, pK:, and concentrations of conjugate acid-base pairs. A7) H=pK, + I pH=pK, 08[[“ A You are given the concentration of the acid (HC:H;O3, 0.35 M) and the conjugate base (NaC2H:O,, 0.55 M) in the question. In addition, you are given the pK, of the acid (HC;H30,, 4.74). These values are entered into the Henderson-Hasselbalch equation and pH is determined: PH =474+ k,g(%] pH=474+ log(1.57) 155
Your preview ends here
Eager to read complete document? Join bartleby learn and gain access to the full version
  • Access to all documents
  • Unlimited textbook solutions
  • 24/7 expert homework help
Acids and Bases ACS V Exams pH=4.74+020=494 which is choice (C) Finally, you can check this answer to make sure it is reasonable. Because you have a higher concentration of conjugate base compared to the acid ([NaC;H;0:] > [HC:H:0,)), you would predict the pH would be higher than the pK; (which it is). Choice (A) is not correct because it is determined using the inverse of the conjugate base to acid molar ratio in the Henderson-Hasselbalch. Choice (B) is not correct because its value is the value of the pK, for the given acid. Choice (D) is not correct because its value is the value of the pH for a neutral aqueous solution. Practice Questions Related to This: PQ-21, PQ-22, PQ-23, PQ-24, and PQ-25 SQ-8. What volume (in mL) of 0.150 M NaOH(aq) is required to neutralize 25.0 mL of 0.100 M HS0x(aq)? (A) 16.7mL (B) 333mL (C) 66.7 mL (D) 75.0mL Knowledge Required: (1) How to balance the chemical equation for a Brensted-Lowry acid-base reaction. (2) Solution Stoichiometry. Thinking it Through: You are asked in the question to determine what volume of a given solution with a known molar concentration is needed to neutralize a given volume of a solution with a known molar concentration. To begin, it is necessary to write a balanced equation for the acid-base reaction: 2NaOH(aq) + H.S0, (ag) —2H.0(1) +Na.SO, (aq) You note that from the stoichiometric coefficients in the balanced equation, two moles of NaOH(aq) are necessary to neutralize one mole of H,SO.. Next, you recall the relationship between concentration (M), moles (niue). and volume of solution (L): mole(n,,,,. ) = molar concentration x volume Therefore, based on the information in the question, you determine the number of moles of H2SOs present. 25.0 mL W‘.}(L):zso“o-‘ mol H,S0, L 1000 mL h To determine the volume of NaOH needed you usc the stoichiometry from the balanced chemical equation: N 2 ( 250x10° mol H,30, | 2 mNeOH ( L ] 1000 "'L)=33.3 mL Choice (B) 2%\ Vol H,50, | 0.150 mol NaOH | 1L Choice (A) is not correct becausc it was determined using the inverse mole ratio {1:2) instead of (2:1). Choice (C) is not correct because it was determined using a 1:1 mole ratio or omitted the mole ratio. Choice (D) is not correct because it was determined using the concentration of NaOH with the volume of H2S0; and the concentration of H2SO4 when calculating the volume of NaOH. Practice Questions Related to This: PQ-26, PQ-27, PQ-28, PQ-29, and PQ-30 Practice Questions (PQ) M PQ-1. Which substance is acting as a Lewis acid? BFs(g) + F(g)= BF:i(g) (A) BF: (B) F- (C) BF: (D) This is not a Lewis acid-base reaction. 156
Your preview ends here
Eager to read complete document? Join bartleby learn and gain access to the full version
  • Access to all documents
  • Unlimited textbook solutions
  • 24/7 expert homework help
2 ACS Acids and Bases @ Exams PQ-2. What is a true statement about Lewis acids and bases? (A) A Lewis acid must have a nonbonding (lone) pair of electrons. (B) A Lewis base must have a nonbonding (lone) pair of electrons. (C) In a Lewis acid-base reaction, a pair of electrons is donated from the acid to the base. (D) In a Lewis acid-base reaction, one species always goes from having a charge to being electronically neutral. PQ-3. What are the Brensted-Lowry bases in this reaction? NH, (aq) + H,0(aq) = NH; (aq) + OH (aq) (A) NH;and OH~ (B) H:0 and NH4~ (C) NH;and H;O (D) NH;" and OH~ PQ-4. What is the hydroxide ion concentration in an aqueous solution with a pH of 7.0 at 25 °C? (A) 0.0M (B) 1.0x107*M (C) 1.0x107"M D) 7.0M PQ-5. What is the pH of a 0.0050 M solution of Ba(OH)(aq) at 25 °C? (A) 2.00 (B) 230 ©) 11.70 (D) 12.00 PQ-6. The formation of a complex ion such as Cu(NHs)s*"(aq) can best be categorized as a(n) ___ reaction. Cu**(aq) + 4NHs(aq) Cu(NH:)s*(aq) (A) Arrhenius acid-base (B) Bronsted-Lowry acid-base (C) Lewis acid-base (D) oxidation-reduction PQ-7. A 10.0 mL portion of 0.010 M HCl is added to 100.0 mL of water. What is the pH of the resulting solution? (A) between 3.02 and 3.10 (B) between 2.90 and 3.01 (C) between 2.02 and 2.10 (D) between 1.90 and 2.01 PQ-8. The pOH of pure water at 40 °C is 6.8. What is the hydronium concentration, [H;0"], in pure water at this temperature? (A) 1.0x10*M (B) 63x10°M (€©) 1.0x10"M D) 1.6x107M PQ-9. What is the pH of a 0.053 M solution of KOH at 25 °C? (A) 0.89 (B) 128 ©) 1272 (D) 13.11 PQ-10. What is the equilibrium concentration of nitrous acid, HNO; (K, = 4.5%10), in a solution that has a PH of 1.657 (A) 0.0032M (B) 0.022M (€) 049M @) 1.1M PQ-11. The pain reliever codeine is a weak base with a K, equal to 1.6%10°%. What is the pH of a 0.05 M aqueous codeine solution? (A) 7.1 (B) 105 ©) i ) 127 157
Your preview ends here
Eager to read complete document? Join bartleby learn and gain access to the full version
  • Access to all documents
  • Unlimited textbook solutions
  • 24/7 expert homework help
Acids and Bases " ACS Exams A PQ-12. Besides water molecules, what species is/are present at the greatest concentration when NHs(g) is a,.d" bubbled into water? (K for NHa(aq) is 1.8x10°9) (A) NHs(aq) (B) NH:OH(aq) (C) NH.'(aq) and OH-(aq) (D) NH;(aq) and H;O"(aq) PQ-13. Which 0.10 M solution will have the largest concentration of hydroxide ion? (A) NH; (Ky of NH; = 1.8x107) (B) NaCN (K, of HCN =4.9x107'%) (©) NaClO: (Ka of HCIO; = 1.7%10°%) (D) NaHCO:; (Kb of HCO5 = 2.3x10°%) Mv‘,\ PQ-14. Which is the strongest acid in aqueous solution? (A) acetic acid (K, = 1.8x107) (B) benzoic acid (K, = 6.3x10°%) (C) formic acid (K, = 1.7x107%) (D) hydrofluoric acid (K, = 7.1x107%) M,& PQ-15. Which aqueous acid has the largest K, value? (A) HBrO (B) HBrO, (€) HCIO (D) HCIO; s PQ-16. In non-aqueous solution, the trend in acid strength is observed to be: K (HI) > K, (HBr) > K, (HCl) M ‘Which periodic trend best explains this observed pattern? (A) Atomic radius: as the bond between H and the halogen becomes shorter, acid strength decreases (B) Electron affinity: as the halogen becomes more attracted to electrons, acid strength increases (C) Electronegativity: as the bond between the two atoms becomes less polar, acid strength decreases (D) lonization energy: as it becomes harder to remove an electron from the halogen, acid strength increases s PQ-17. The reaction is observed to have a Keg > 1. Il HSO, (ag) + HPO," (ag) = SO, (ag) ~ H,PO, (aq) What is the strongest acid present in this equilibrium? (A) HzPO: (B) HPOZ (C) HSOy (D) SO PQ-18. What is the pH of a 0.400 M sodium formate (NaCHO;) solution? K, (HCHO,) = 1.8x107* (A) 2.07 (B) 533 ©) 867 D) 11.93 w0 PQ-19. Which salt will form a basic aqueous solution? o (A) NaF (B) KBr Licl (D) NH.NO3 PQ-20. The pH of a 0.050 M aqueous solution of ammonium chloride (NH4Cl) falls within what range? (A) 0to2 (B) 2to7 (C) Tto 12 D) 12to0 14 PQ-21. What is the pH of a buffer solution containing equal volumes of 0.11 M NaCH;COO and 0.090 M. CH;COOH? K, (CH;COOH) = 1.8x107° (A) 242 (B) 483 (©) 11.58 (D) 1391 158
Your preview ends here
Eager to read complete document? Join bartleby learn and gain access to the full version
  • Access to all documents
  • Unlimited textbook solutions
  • 24/7 expert homework help
s A CS Acids and Bases Exams PQ-22. Which pair of compounds would be the best choice to make a buffer solution with a pH around 5? (A) HF and NaF K. =6.9%10* (B) C;HsCO;H and C;HsCO:Na Ky =1.3x107* (C) HCIO and NaClO K,=238x10% (D) NHClandNH; K, =5.6x107"° PQ-23. Which pair could be used to make a buffer solution? (A) acetic acid and sodium chloride (B) acetic acid and sodium hydroxide (C) hydrochloric acid and potassium chloride (D) hydrochloric acid and sodium hydroxide PQ-24. What is the pH of a buffer solution made by adding 0.010 mole of solid NaF to 50. mL of 0.40 M HF? Assume no change in volume. K, (HF) = 6.9x107* (A) 1.6 ®) 19 ©) 29 (D) 32 PQ-25. An aqueous buffer solution contains only HCN (pK, = 9.31) and KCN and has a pH of 8.50. What can be concluded about the relative concentrations of HCN and KCN in the buffer? (A) [HCN]>[KCN] (B) [HCN] <[KCN] (€) [HCN]=[KCN] (D) nothing can be concluded about the relative concentrations PQ-26. Methyl orange is an indicator that changes color from red to yellow-orange over the pH range from 2.9 to 4.5. Methyl orange would be the most appropriate indicator for which type of acid -base titration? (A) A weak acid titrated with a strong base (B) A weak base titrated with a strong acid (C) A strong acid titrated with a strong base (D) Methy! orange would not be an appropriate indicator for any acid-base titration. f PQ-27. Which graph best represents the titration of ammonia with hydrochloric acid? e h £ i, (A) (B) o o volume HCI added ‘volume HC1 added 14| " 7 27 ©) (D) N o volume HC1 added volume HCI added 159
Your preview ends here
Eager to read complete document? Join bartleby learn and gain access to the full version
  • Access to all documents
  • Unlimited textbook solutions
  • 24/7 expert homework help
Acids and Bases - ACS W Exams PQ-28. In the laboratory, 50 mL of 0.1 M HCl is mixed with 50 mL of 0.1 M NaOH and the solution is stirred gently. At equilibrium, what ionic species, if any, will be present in large amounts in the reaction mixture? (A) Na'and CI- only (B) H;O'and OH only (C) H;O0',OH-,Na', and CI- (D) No ions will be present PQ-29. What is the approximate pK, of the weak 1 acid in the titration curve? N - z ¢ s P 2 o 0 2 W0 e volume NaOH added (mL) (A) 2.0 (B) 48 (©) 88 ®) 11.0 PQ-30. What is the pH of a solution that results from mixing 25.0 mL of 0.200 M HA with 12.5 mL of 0.400 M NaOH? (Ka = 1.0x107) (A) 294 (B) 4.94 (©) 9.06 (D) 11.06 160
Your preview ends here
Eager to read complete document? Join bartleby learn and gain access to the full version
  • Access to all documents
  • Unlimited textbook solutions
  • 24/7 expert homework help
[k ol M rOoOw POXIANB LS TODPATN>E > ~"ACS Acids and Bases Qvamm Answers to Study Questions RS wO»T Answers to Practice Questions 11.B 21.B 12.A 22.B 13.B 23.B 14.D 24.C 15.D 25.A 16. A 26.B 17.C 27.A 18.C 28.A 19.A 29.B 20.B 30.C 161
Your preview ends here
Eager to read complete document? Join bartleby learn and gain access to the full version
  • Access to all documents
  • Unlimited textbook solutions
  • 24/7 expert homework help
Acids and Bases 162
Your preview ends here
Eager to read complete document? Join bartleby learn and gain access to the full version
  • Access to all documents
  • Unlimited textbook solutions
  • 24/7 expert homework help
Chapter 13 Solubility Equilibria Chapter Summary: This chapter will focus on the solubility of solids in aqueous media and other solvents. Specific topics covered in this chapter are: * Molar solubility * Equilibrium expressions for solubility reactions * Solubility product constants (Ks;) and related calculations e Common ion effect « Formation reactions, expressions and constants (Kr) Previous material that is relevant to your understanding of questions in this chapter includes: * Solutions and Aqueous Reactions, Part 1 (Chapter 5) * Solutions and Aqueous Reactions, Part 2 (Chapter 9) e Equilibrium (Chapter 11) Common representations used in questions related to this material: Name Example Used in questions related to Compound units mol-L™! molar solubility Macroscopic diagrams ccommon ion effect Where to find this in your textbook: The material in this chapter typically aligns to “Aqueous Equilibria” in your textbook. The name of your chapter(s) may vary. Practice exam: There are practice exam questions aligned to the material in this chapter. Because there are a limited number of questions on the practice exam, a review of the breadth of the material in this chapter is advised in preparation for your exam. How this fits into the big picture: The material in this chapter aligns to the Big Idea of Equilibrium (8) as listed on page 12 of this study guide. Study Questions (SQ) SQ-1. What is the K, expression for MgCOs in water? _ Mg J[cor] x —Me"Iicor] @A) " [MgCOH,0] ® 0] © K,=Mg"][coi] () K»=IMgCO,][H,0] Knowledge Required: (1) How to write a solubility equilibrium expression for a given species in water. Thinking it Through: Y ou are asked in the question to write the equilibrium expression that corresponds to the Kep, solubility product constant, for a given species. The K expression is the concentrations of product(s) raised 163
Your preview ends here
Eager to read complete document? Join bartleby learn and gain access to the full version
  • Access to all documents
  • Unlimited textbook solutions
  • 24/7 expert homework help
Solubility Equilibria c:ACS Exams to the power of their respective stoichiometric coefficients from a balanced chemical equation divided by the | concentrations of reactant(s) raised to the power of their respective stoichiometric coefficients. Solids and liquids are not included in the expression because their concentrations are assumed to be constant. You, therefore, begin by constructing the balanced equation for the dissociation of MgCOsx(s) in water: MgCO; (s) = Mg (aq) + CO3"(aq) From the balanced equation, you can construct the K, expression: K, = [Mg"][CO>"] Choice (C) Choice (A) is not correct because it includes the concentrations of the solid and the liquid. Choice (B) is not correct because it includes the concentration of the liquid. Choice (D) is not correct because it includes the concentrations of the solid and the liquid, and does not include the concentrations of the dissolved ions. Practice Questions Related to This: PQ-1 and PQ-2 SQ-2. Given the two reactions: Cr(OH),(s) == Cr'" (aq) + 30H (aq) K= 1.6¥10°° Cr**(aq) + 4OH™(aq) = [Ct(OH), T (aq) K;=8.0x10% What is the value of X for the reaction? Cr(OH),(s) + OH™ (aq) = [Cr(OH), ]" (aq) (A) —8x10% (B) 2.0x10€ (©) 13x10° D) 5.0%10° Knowledge Required: (1) How to write an equilibrium expression. (2) How to combine multiple chemical reactions and their respective equilibrium constants to form a new reaction and equilibrium constant. Thinking it Through: You are asked to determine the value of the equilibrium constant for a chemical reaction using two reactions with known constants, K, and K. You begin by writing the two equations and corresponding equilibrium constants. You observe that the target equation is the sum of the two given equations. You also then know that when two equations are summed, the corresponding equilibrium constant (K) is the product of the two equilibrium constants (K, and K): Cr(OH), (s) = " (a0) +30H (aa) K, =[Cr" J[on | _[[ertomT ] [ Jlon ] [[er(on).T] Cr'* (aq) +40H" (aq) = [Cr(OH), ] (aq) K; Cr(OH), (s) +OH" (aq) = [ Cr(OH), | (aq) K [[CI(OH)‘]-] . K=[crJ[on | xe—— "=k _xkK, [ ][ ] [Cri‘][OH‘]‘ » oo When reactions are summed, the new K = K, x Kr= 1.3 or 1.3x10°, Choice (C). Choice (A) is not correct because it is the value obtained when K is substracted from Kip. Choice (B) is not correct because it is the value obtained when Kyp+ Kr. Choice (D) is not correct because it is the value obtained when K¢+ K. Practice Questions Related to This: PQ-3 SQ-3. What is the solubility product constant, Ks,, of Mg(OH): if its molar solubility in water is 1.6x10* mol-L1? (A) 2.6x10°% (B) 1.6x107! (C)_1.0x1072 (D) 4.1x10°"2 164
Your preview ends here
Eager to read complete document? Join bartleby learn and gain access to the full version
  • Access to all documents
  • Unlimited textbook solutions
  • 24/7 expert homework help
*ACS Solubility Equilibria @' Exams Knowledge Required: (1) Definition of molar solubility. (2) How to write a solubility equilibrium expression. (3) How to determine a solubility product constant (Kp) given molar solubility. Thinking it Through: You are asked in the question to determine the solubility product constant () for a given species with a known solubility in water. To begin, you construct a balanced chemical equation for the dissociation and a corresponding Ky, expression: Mg(OH), (s) = Mg" (aq) + 20H"(aq) K, = [Mg"][OH | You see that you are given the “molar solubility” of Mg(OH); in the question. To understand what this is, you proceed to set up the initial-change-equilibrium (ICE) table for this process where you have purposely included the states to remind yourself that Mg(OH)x(s) is not included in the K expression: Mg(OH)y(s) = Mg*(aq) + 20H(aq) Initial - 0 0 Change - +x +2x Equilibrium - x 2x From here you see that the molar solubility (the value of “x” in the table) corresponds to the number of moles solute in 1.0 L of a saturated solution or [Mg(OH)z]. Note: You know this is used to standardize the comparison of solubility of various ionic compounds where the fon ratios are not equivalent (such that you can compare [AI(OH)s] or the molar solubility of AI(OH)s to [AgOH] or the molar solubility of AgOH but cannot compare the respective concentrations of the hydroxide ion to predict which compound is more soluble). Based on the coefficients from the balanced chemical equation, the solubility concentration has a 1:1 ratio with the concentration of Mg** and a 1:2 ratio with the concentration of OH". You can thus enter these values into the Ks, expression and solve: K, =[Mg* J[OH™ | =(x)(2x) =4x K, =[16x10*][2x(1.6x10%)] =1.6x10"" Choice (A) is not correct because the value is obtained using a Ky, expression that does not account for the coefficient of hydroxide ion in the balanced chemical equation. Choice (C) is not correct because the value is obtained using the inverse of the 1:2 mole ratio when calculating the concentration of hydroxide ion. Choice (D) is not correct because the value is obtained without using the 1:2 mole ratio when calculating the concentration of hydroxide ion. Choice (B) Practice Questions Related to This: PQ-3, PQ-4, PQ-5, PQ-6 and PQ-7 $Q-4. _ Which compound has the highest molar solubility? (A) Agl, Ky =8.52x10"7 (B) BaCOy, Kyp=2.58x10° (C) Fe(OH);, Kip=2.79%10 (D) ZnS, Ky =3.00x10°% Knowledge Required: (1) Definition of molar solubility. (2) How to write a sol\fiaili}y Eqv ibrium ex}:réssion. (3) How to determine molar solubility given a solubility product constant (Kp). Thinking it Through: You are asked in the question to evaluate a series of species and their provided K, values to determine which species has the highest molar solubility. To make this evaluation, you consider each species. To begin, you write the balanced equation and the corresponding K., expression for each. Choice (A): Agl(s) = Ag"(aq)+1"(aq) KXP=[Ag'][I’] Choice (B): BaCO, (s) = Ba®" (aq)+ CO:" (aq) K,=[Ba*][cor] Choice (C): Fe(OH), (s) = Fe" (aq)+30H™ (aq) K,,=[Fe* JJOH' ]’ 165
Your preview ends here
Eager to read complete document? Join bartleby learn and gain access to the full version
  • Access to all documents
  • Unlimited textbook solutions
  • 24/7 expert homework help
Solubility Equilibria ACS ¥ Exams Choice (D): ZnS(s) = Zn* (aq) +S* (aq) K,,=[zn™ |[s* ] Next, you assume that equilibrium is reached and the solution is saturated. Additionally, you assume there is not a starting concentration of any species. The variable x will be used to represent one mole equivalent of the solid (for example, Agl) when expressing the concentrations of each species, so then x represents the molar solubility. Therefore, when x values and the K, values are substituted for each, you obtain these expressions: Choice (A): 8.52x107 = x-x Choice (B): 2.58x107 = x-x Choice (C): 2.79x10™" = x-(3x)’ Choice (D): 3.00x107 = x-x For each answer option, you solve for x; because x cannot be negative, only positive values of x are retained. Choice (A): 9.2x107° Choice (B): x= 5.1x107 Choice (C): 1.0x10™"° Choice (D): x= 5.5x10™* Therefore, the species with the highest molar solubility (the highest value of x) is BaCOs, Choice (B). Practice Questions Related to This: PQ-8, PQ-9, PQ-10, PQ-11, PQ-12, PQ-13, PQ-14, PQ-15, PQ-16, and PQ-17 SQ-5. A solution contains 0.002 M Pb*" and 0.002 M Ag”. What happens when Ksp_ NaCl(s) is added to bring the chloride ion concentration to 0.01 M? PbCly 1.2x10°° AgCl_ 1.8x10° (A) Neither PbCL nor AgCl will precipitate. (B) Both PbCl: and AgCl precipitate. (C) Only PbCl; precipitates. (D) Only AgCl precipitates. Knm’vledige Required: (1) How to write a solubility equilibfium expression. (2)’ How to determine a reaction quotient, Q. (3) How to predict precipitation by comparing O with K. Thinking it Through: Y ou are asked in the question to determine what occurs when a concentration of a given ion is brought to a certain level. To do this you will calculate the value of O for each set of conditions and compare to the value of K. The criteria for the comparison are: If Qsp < Kip the reaction will proceed to form more products: no precipitate forms. Using AgCl as the example, the concentration of the ions is sufficiently small that the solution is unsaturated and the reaction will shift to the right: AgCl(s) = Ag” (aq)+Cl " (aq) If Qsp > Kip the reaction will proceed to form more reactants: a precipitate forms. Using AgCl as the example, the concentration of the ions is sufficiently large that the solution is saturated and the reaction will shift to the left: AgCl(s) < Ag’ (aq)+Cl (aq) 1f Qsp > Kip the reaction is at equilibrium: the solution is just saturated. Using AgCl as the example, the concentration of the ions is at the saturation point such that the solution is saturated and the reaction is at equilibrium: AgCl(s) = Ag’ (aq)+Cl (aq) Therefore, for this question, the relevant O expressions are: AgCl(s) = Ag’ (aq)+Cl (aq) Q= [Ag' ]0 [Cl' ]0 PbCI, (5) = Pb*" (aq)+ 2C1" (aa) 0 =[Pb* ] [CI |} For PbCly: 0=(0.002)(0.01)° =2x10" For AgCl: 0=(0.002)(0.01)=2x10" The Q for PbCl, is less than Ky, for PbCls, so PbCl> will net precipitate. 166
Your preview ends here
Eager to read complete document? Join bartleby learn and gain access to the full version
  • Access to all documents
  • Unlimited textbook solutions
  • 24/7 expert homework help
ACS Solubility Equilibria Exams The Q for AgCl is greater than K, for AgCl, so AgCl will precipitate. Choice (D) is the correct answer. Practice Questions Related to This: PQ-18 and PQ-19 SQ-6. A sample of “hard” water contains about 2.0x102 mol of Ca?* ions per liter. What is the maximum concentration of fluoride ion that could be present in hard water? Assume fluoride is the only anion present that will precipitate calcium ion. (Ksp of CaFa(s) at 25 °C = 4.0x10°1) (A) 2.0<10°M (B) 22x10°M (©) 44x10°M (D) 2.0x102M Knowledge Required: (1) How to write a solubility equilibrium expression. (2) How to determine the concentration of one ion given the solubility product constant (Kp) and concentration of a common ion. Thinking it Through: You are asked to determine the maximum amount of a species that can be dissolved into a solution that contains a common ion with the species being dissolved. You begin by constructing a balanced chemical equation and K, expression for the dissolution: CaF,(s) = Ca™ (aq) + 2F"(aq) = [Ca™J[FJ Next, you consider what this looks like if CaF; was first in pure water. By slowly adding solid CaF; to water, initially (when [Ca®"] and [F-] are very small), the solution would remain homogeneous. After some point, the concentration of the ions reaches a concentration such that the solution is saturated and no more solid CaF; will dissolve, and a heterogeneous solution has formed. You know this is called the “molar solubility” and determine this using the process outlined in SQ-3. Using this, you find the molar solubility for CaF; in pure water is equal to 2.2x10~* M or in pure water, [Ca?"] = 2.2x10* M and [F] = 4.4x10* M. Pure water [Ca*] <22x10%4M [Ca*]>22x10% M [Ca¥]=0M [F]<44x104M [F]>44x10* M [F1=0M solution is unsaturated solution is saturated and is homogeneous and is heterogeneous & > Add Can(s) Contmue to slowly add CaFy(s) slowly Now, you consider what this looks like for CaF; in tap water containing [Ca’*} = 2.0x102 M. By slowly adding solid CaF to tap water, initially (when [F] is still very small), the solution would remain homogeneous. After some point, the concentration of the fluoride reaches a concentration such that the solution is saturated and no more solid CaF; will dissolve, and a heterogeneous solution has formed. However, this is different because you started with [Ca>*}o. So to determine the concentration of the fluoride ion, [F-], you will include the initial concentration of the calcium ion: Tap water [Ca*]=2.0x102M [Ca**]=2.0x102M [Ca*]o=2.0x102M | [F]<? [F]>7M [F]=0M solution is unsaturated solution is saturated and is homogeneous and is heterogeneous S - Add CaF 2(s) Continue to slowly add CaFa(s) slowly 167
Your preview ends here
Eager to read complete document? Join bartleby learn and gain access to the full version
  • Access to all documents
  • Unlimited textbook solutions
  • 24/7 expert homework help
Solubility Equilibria . ACS CaF(s) = Ca*(aq) + 2F (aq) Initial - 2.0%102 0 Change - +x 2x Equilibrium - 2.0%107 +x I Because the concentration of the calcium ion is higher, you can assume the change in the concentration of the calcium ion (from the added CaFs) will be sufficiently small (or assume 2.0x107 +x = 2.0x10™) to be unchanged from the initial concentration: k,=[ca J[F T =[ca"](2x) =[Ca* Jaxr® ar=—2= [Fl=2x=2(22x10")=44x10" M Corresponding to choice (C). Choice (A) is not correct because it is the value of x if the coefficient of “2” for F~ was not considered in the K expression and initial-change-equilibrium table. Choice (B) is not correct because it is the value of x; however, the final concentration of F~ is 2-x. Choice (D) is not correct because it is the initial concentration of Ca’* and the approximate equilibrium concentration of Ca®". Practice Questions Related to This: PQ-20 What will change the value of K, for silver azide (AgN;)? A3 M (A) Adding water to the solution. (B) Adding silver ions to the solution. (C) Removing azide ions from the solution. (D) Increasing the temperature of the solution. Knowledge Requirt;l: (1) What influences the value of Ksp Thinking it Through: Y ou are asked in the question to evaluate what changes to a system at equilibrium would result in a change in the value of Ks,. Choice (A) would result in decreasing concentrations and thus increases in the amount of silver azide dissolved. Choice (B) would result in a common ion effect whereby the addition of silver ions would lead to more silver azide precipitation. Choice (C) would have the opposite effect whereby the removal of azide ions would result in more silver azide being dissolved. Choices (A), (B), and (C) impact Q for the dissolution, but do not impact the value of K;. Choice (D), though, does have a direct impact on the value of K., whereby an increase in temperature results in a different K, value. Practice Questions Related to This: PQ-21, PQ-22, PQ-23, PQ-24, PQ-25, PQ-26, PQ-27, PQ-28 and PQ-29 8. SQ When comparing O and K5, of an unsaturated solution, . (A) O<Ky B) 0=Ky ©) 0>Ks (D) 0=Ky=0 Knowledge Required: (1) The definition of 0. (2) How O and K. are related. Thinking it Through: Y ou are asked in the question to determine the relationship between Q and Ky, for an unsaturated solution. Recall that for an unsaturated solution, more starting material can be dissolved into solution. Therefore, the molar solubility has not been reached. The solubility product constant, Ks,, is defined as the product of the maximum concentration of dissolved species, i.e. the molar solubility. The value of O is defined as the product of the actual concentrations of dissolved species. Therefore, if O is greater than K, there are concentrations of the dissolved species greater than the maximum allowable concentrations and thus species will precipitate out until the concentrations of dissolved species matches Ks,, Choice (C). If Q is equal to K, there are exactly the concentration of dissolved species that match Ks;, Choice (B). If Q is less than K, there are concentrations of the dissolved species that are less than the maximum concentrations (or molar solubility values) and thus more species could be dissolved into solution, Choice (A). Choice (D) would only be true if the species is 168
Your preview ends here
Eager to read complete document? Join bartleby learn and gain access to the full version
  • Access to all documents
  • Unlimited textbook solutions
  • 24/7 expert homework help
cACS Solubility Equilibria entirely insoluble. An unsaturated solution is a solution whereby more species could be dissolved; therefore, Choice (A) where O is less than K, is correct for an unsaturated solution. Practice Questions Related to This: PQ-30 Practice Questions (PQ) PQ-1. What is the K, expression for CoCOs(s) in water? @) g - [ComNCOT] ®) g -[CoMICOT] ¥ [CoCO,][H,0] N [H.0] ©) K, =[Co™][COT] D) K, =[CoCO;]H,0] PQ-2. What is the K, expression for Nis(POxs)a(s) in water? A) K, =INiT[POIF ®) K, =INi"J[POIT K, =(3xINi")(2x(POL ) ® K, =(3xNi) (2x(POS ) PQ-3. Given the two reactions: wn"“‘ CuCl(s) + CI" (aq) = [CuCl, ] (aq) K=52x10" Cu’(aq) +2CI" (aq) = [CuCl, ] (aq) ‘What is the value of K, for CuCI? (&) KxKr ® £ © K o ! K, X KxK, .0x10° PQ-4. A saturated solution of MgF; contains 1.6x10~* mol of MgF; per liter at a certain temperature. What is the K, of MgF> at this temperature? (A) 2.7x10°¢ (B) 1.6x10°¢ (©) 3.1x107° (D) 62x10° PQ-5. A saturated solution of AI(OH); has a molar solubility of 2.9x10 M at a certain temperature. What is the solubility product constant, Ky, of AI(OH); at this temperature? (A) 2.1x10% (B) 1.9x10% (C) 84x107® (D) 2.5x107"7 PQ-6. Silver chromate (Ag:CrOs; Kp = 1.1%107'2) has a molar solubility of 6.5x10" M. What calculation provides the concentration of silver ions in a saturated solution of Ag,CrO4? (A) 2% (65%109M (B) 1% (65<109M (C) [2%(65x10%FM (D) {1x(1.1x107) M PQ-7. The solubility of lead(Il) iodide in water is 0.62 grams per 1.0 L of solution. What is the solubility product constant, Ky, for the dissolution? Pbl, (s) = Pb* (aq) + 2I"(aq) (A) 2.4x10° (B) 9.7x107 (©€) 1.8x10¢ (D) 4.0x10° PQ-8. For Ag:S the solubility product constant, Ks, is 6x107*. What is the molar solubility of Ag.S? (A) 1x107"°M (B) 2x10"°M (©€) 210 M (D) 2x10°°M PQ-9. For BaF; Ky, = 1.0x10°. What is the molar solubility of BaF: in a solution containing 0.10 M NaF? (A) 63x10°M (B) 1.0x10*M (©) 2.5x10°M (D) 1.0x10° M 169
Your preview ends here
Eager to read complete document? Join bartleby learn and gain access to the full version
  • Access to all documents
  • Unlimited textbook solutions
  • 24/7 expert homework help
Solubility Equilibria s ACS Exams M PQ-10. Which saturated solution has the highest concentration of F- ions at 25 °C? (A) BaFy, Ky, = 1.8x107 (B) CaFy, Kyp=3.5x107" (€) MgFy, Kip=7.4x107"1 (D) StFa, Kip=25%10° PQ-11. What is the molar solubility of AgsPO4 in water? K, (AgsPOs) = 1.4x107'6 (A) L1x10*M (B) 4.8x10°M (€) 52x10°M (D) 6.8x10°M PQ-12. Given Ki; of CaF3 is 3.5%107"". The molar solubility of CaF in 0.200 M NaF is (A) 2.1x10* M (B) 1.8x107°M (C) 8.8x107"°M D) 35x10""M PQ-13. What is the maximum mass of solid barium sulfate (233 g-mol™') that can be dissolved in 1.00 L of 0.100 M Na>SOx solution? Ky, (BaSOs) = 1.5x10° (A) 1.5%10%¢g (B) 2.1x10%g (C) 3.5%x10%¢g (D) 9.0x10%¢g PQ-14. Rank the following species from most to least soluble in water. I Ba(NOs),, Kip = 4.64x10° IL Cd(OH);, Kip = 7.20x1075 . FeCOs, Kp=3.13x10"" (A) 1> 11>11 ®) 1> m>1n (©) nm>m>1 D) 1>1>Mm PQ-15. Rank the following species from least to most soluble in water. L PbBr;, Ky =6.60x10" IL PbCly, Kyp= 1.70%10° M. Pbl, Ky =9.80x107 (A) I<Hm<I ®B) m<m<i © m<I<un (D) HI<II<I PQ-16. For LiF K, =1.84x107. What is the molar solubility of LiF? (A) 6.78x10" M (B) 429x102M (©) 8.78x10°M (D) 3.00x10°M c#d,d PQ-17. Which saturated solution has the highest concentration of OH™ ions at 25 °C? (A) AI(OH);, Ky =3.0x107 (B) Ca(OH), K = 5.02x10 (C) Cd(OHY;, Kyp=7.2x107 (D) Pr(OH);, Ky =3.4x107 PQ-18. The solubility of solid silver chromate, Ag.CrOs, which has a K;; equal to 1 pure water 9.0x107'2 at 25 °C, is determined in water and in two different aqueous Il 0.1 MAgNO; solutions. Predict the relative solubility of silver chromate in the three I 0.1 M Na,CrOq solutions. (A) I=n=11 (B) I<H<uI ©) m=m<I (D) m<u<t PQ-19. What will be the result if 100 mL of 0.06 M Mg(NO5) is added to 50 mL of 0.06 M NaxC,04? Assume the reaction is taking place at 25 °C. (K, of MgCy04(s) at 25 °C = 8.6x10°%) (A) No precipitate will form. (B) A precipitate will form and an excess of Mg?* ions will remain in solution. (C) A precipitate will form and an excess of C;04>" ions will remain in solution. (D) A precipitate will form and both Mg?* and C,04?" ions are present in excess. 170
Your preview ends here
Eager to read complete document? Join bartleby learn and gain access to the full version
  • Access to all documents
  • Unlimited textbook solutions
  • 24/7 expert homework help
§ ACS Solubility Equilibria Exams PQ-20. A solution contains 0.002 M Be** and 0.002 M Ca?'. What happens when Ky NaOH(s) is added to bring the hydroxide ion concentration to 0.10 M? Be(OH): 6.9%10% Ca(OH): 5.0x10° (A) Neither Be(OH); nor Ca(OH), will precipitate. ~ (B) Both Be(OH), and Ca(OH), precipitate. (C) Only Be(OH), precipitates. (D) Only Ca(OH); precipitates. w‘d’,\ PQ-21. The addition of solid Na;SOx to an aqueous solution in equilibrium with solid BaSOx will cause (A) no change in [Ba®"] in solution. (B) more BaSO: to dissolve. (C) precipitation of more BaSOx. (D) an increase in the K.y of BaSOx. PQ-22. If solid NasPOx is added to a saturated aqueous solution of Cas(POa)z, what will happen to the "afid‘\ concentration of Ca** in solution? (A) It will decrease. (B) It will increase. (C) It will be unchanged. (D) It will become equal to the PO~ concentration. A PQ-23. AgBr has very low solubility in water. Silver ions form a complex ion with thiosulfate. o AZ (1) 25,0} 1) = [AR(5.0,), " 19) If sodium thiosulfate is added to a saturated solution of AgBr in equilibrium with solid AgBr, how will the concentrations of free Ag'(aq) and Br-(aq) change? (A) Ag'(aq) decreases, Br(aq) decreases (B) Ag’(aq) decreases, Br(aq) increases (C) Ag'(aq) increases, Br(aq) decreases (D) Ag'(aq) increases, Br(aq) increases A PQ-24. For which slightly soluble substance will the addition of perchloric acid to its solution have no effect M on its solubility? (A) AgBr(s) (B) Cu(OH)(s) (€) MgCOs(s) (D) PbFay(s) PQ-25. The solubility of solid copper(I) chloride, CuCl, which has a K, L pure water equal to 1.72x107 at 25 °C, is determined in water and in two 1L 0.1 MNaCl different aqueous solutions. Predict the relative solubility of 0L 0.1 M MgCly copper(I) chloride in the three solutions (A) I<I<In (B) 1<mI<n ©) m<m<I1 D) m<m<r A PQ-26. The addition of solid sodium hydroxide (NaOH) to an aqueous solution in equilibrium with solid o™ Me(OH): will cause (A) no change in [OH"] in solution. (B) more Mg(OH), to dissolve. (C) adecrease in [Mg*"] in solution. (D) an increase in the Ky, of Mg(OH),. A PQ-27. Which will change the value of K, for mercury(I) sulfate (HgSO4)? M (A) adding Hg" ions to the solution. (B) removing SO4* ions from the solution. (C) decreasing the temperature of the solution. (D) decreasing the pH of the solution. 171
Your preview ends here
Eager to read complete document? Join bartleby learn and gain access to the full version
  • Access to all documents
  • Unlimited textbook solutions
  • 24/7 expert homework help
Solubility Equilibria oACS @ Exams PQ-28. For Agl K, =8.5%107 and for [Ag(CN):] K;= 1.0<10%'. Which statement is true? M (A) Agl is completely insoluble in a solution that contains CN~. (B) Agl s less soluble in water than in a solution that contains CN . (C) Agl is more soluble in water than in a solution that contains CN-. (D) Aglis equally soluble in water and in a solution that contains CN . PQ-29. Ammonia (NH;) is slowly added to a saturated solution of Ag:CO; (Kip = 8.46x107'2). Which statement s true if K; for [Ag(NH3)z]" is 1.6%107 (A) Ag>CO; is completely miscible in a solution that contains NHa. (B) AgCOs is more soluble in water than in a solution that contains NH;. (C) Ag;COs is less soluble in water than in a solutjon that contains NHs. (D) Ag:CO;s is equally soluble in water and in a solution that contains NHs. M‘d PQ-30. When comparing O and K, of a saturated solution, . (A) 0<Kqp (B) O=Ks ©) Q=K D) 9=K3=0 172
Your preview ends here
Eager to read complete document? Join bartleby learn and gain access to the full version
  • Access to all documents
  • Unlimited textbook solutions
  • 24/7 expert homework help
El ol wEAN Sexnpmsprn >EFREPTIE SO > ACS Solubility Equilibria @ Exams Answers to Study Questions i »oN0D Answers to Practice Questions 11.B 21.C 12.C 22.A 13.C 23.B 14.A 24.A 15.C 25.D 16.B 26.C 17.B 27.C 18.D 28.B 19.D 29.C 20.B 30.B 173
Your preview ends here
Eager to read complete document? Join bartleby learn and gain access to the full version
  • Access to all documents
  • Unlimited textbook solutions
  • 24/7 expert homework help
Solubility Equilibria 174
Your preview ends here
Eager to read complete document? Join bartleby learn and gain access to the full version
  • Access to all documents
  • Unlimited textbook solutions
  • 24/7 expert homework help
Chapter 14 Thermodynamics Chapter Summary: This chapter will focus on entropy and free energy, energy, and enthalpy. The transfer of energy during chemical and physical changes will be calculated. The usefulness of entropy and free energy being state functions will be used to calculate the values of entropy and free energy changes using Hess’s law. Specific topics covered in this chapter are: Entropy and trends in molar entropy values Predicting the sign of the entropy change for a process Calculating the value of AS for a chemical reaction Definition of a spontaneous process Calculating AG for a reaction using Hess’s law Predicting the temperature dependence of spontaneity Relating values of AG® and equilibrium coefficients Previous material that is relevant to your understanding of questions in this chapter include: Significant figures (Toolbox) Scientific notation (Toolbox) Balancing chemical reactions (Chapter 3) Heat and Enthalpy (Chapter 6) Equilibrium (Chapter 11) Common representations used in questions related to this material: Name Example Used in questions related to compound units kJ-mol™' and J-mol K™ density, enthalpy changes for reactions (AH), Gibbs free energy changes for reactions (AG), entropy changes for reactions (AS) Particulate diagrams ol . ° © g entropy changes © ol ® o «© O o Pred Where to find this in your textbook: The material in this chapter typically aligns to “Thermodynamics”. The name of your chapter may vary. Practice exam: There are practice exam questions aligned to the material in this chapter. Because there are a limited number of questions on the practice exam, a review of the breadth of the material in this chapter is advised in preparation for your exam. How this fits into the big picture: The material in this chapter aligns to the Big Ideas of Energy and Thermodynamics (6) and Equilibrium (8) as listed on page 12 of this study guide. 175
Your preview ends here
Eager to read complete document? Join bartleby learn and gain access to the full version
  • Access to all documents
  • Unlimited textbook solutions
  • 24/7 expert homework help
Thermodynamics o ACS ¥ Exams Study Questions (SQ) SQ-1. Of the four listed compounds, three have correct values for molar entropy, S°, associated with them. M For which species is the entropy value incorrect at 298 K? Substance §°(J-mol-K™') Ay Cas) 202 (B) Br(l) 152 © COAe) 214 (D) AlICly(s) 109 Knowledge Required: (1) Trends in molar entropy values. Thinking it Through: You need to identify the incorrect pairing of species and molar entropy values. You remember the general trends for molar entropy values: (1) The more restricted the motion of the physical state, the lower the entropy value, $°%(s) < S%(1) < S%(g) (2) The more complex the species, more atoms/ions present, the larger the value of S°. (3) As temperature increases, the value of §° increases. You would expect the values of the solids to be less than the values of the liquids and these would be less than the values of the gases. Using this you predict that the values for Ca(s) and AIClx(s) should be smaller than the value of Bra(1) and the value for CO: should be the largest. You also know that the value of for Ca(s) should be smaller than the value for for AICI(s). You predict the values should go in the order S%(Ca(s)) < SYAICI(s)) < S%Bra(1)) < S(COx(g)) ‘When you substitute the given values into the prediction you get: 202<109<152<214 It is clear that the value for Ca(s) is the incorrect one. Choice (A) is the incorrect pair, and therefore the correct answer. The other species and molar entropy values are correctly paired up. Practice Questions Related to This: PQ-1 and PQ-2 SQ-2. Which change is likely to be accompanied by an increase in entropy? (A) Na(g) +3Ha(g) 2NHy(g) (B) Ag'(aq) +Cl(aq) AgCI(s) ©) COx(s) COx(g) (D) H:0(g) H:0(1) Knowledge Requifed: (li Definition of entropy. (2) Trends in molar entropy values. Thinking it Through: You recall that the entropy change for a process is equal to: AS = §°(products) S°(reactants) You also know that for an increase in entropy you are looking for a process with a positive value of AS (AS > 0), this occurs when the entropy of the products is greater than the entropy of the reactants: AS>0 §° (products) - (reactants ) > 0 §° (products) > (reactants) You consider each choice and predict the sign of AS for each. In choice (A) you have: Reactants (4 moles gas) Products (2 moles gas) You predict the sign of AS for this process to be (-) because the Angas =~2 or a decrease in entropy. 176
Your preview ends here
Eager to read complete document? Join bartleby learn and gain access to the full version
  • Access to all documents
  • Unlimited textbook solutions
  • 24/7 expert homework help
. ACS Thermodynamics Exams I*n"ch;ic?(fi) i'ou have: Reactants (2 moles éolufion)» Products (l-mole solid) You predict the sign of AS for this process to be () because the transition from solution to solid results in a decrease in entropy. In choice (C) you have: Reactants (1 mole solid) - Products (I mole gas) You predict the sign of AS for this process to be (+) because Angs = +1 or an increase in entropy. In choice (D) you have: ~ Reactants (1 mole gas) Products (1 mole liquid) You predict the sign of AS for this process to be () because Az =—1 or a decrease in entropy. The correct choice is (C). Practice Questions Related to This: PQ-3, PQ-4, P-5, PQ-6, PQ-7, and PQ-8 $Q-3. For this reaction at 25 °C, AH® = ~1854 kJ'mol™' and AS®=-236 J-mol™""K™' . CH;COCHj(g) + 402(2) 3COu(g) + 3H:0() What is the value of AG® for this reaction? (A) -1784kJ'mol" (B) -1848kI'mol! (C) -1924kJ'mol' (D) 68500 kJ-mol" Knowledge Required: (1) The calculation of free energy changes from enthalpy and entropy data. (2) The correct use of units. Thinking it Through: You recognize that you are given a temperature and values for AH° and AS® and are being asked to calculate AG®. You know to use the relationship: AG® = AH®—TAS®. You also recall that the units of AH® are in k] and the units of AS® are in J, so you must convert. The temperature you are given is in °C and you must use the value expressed in kelvin. You begin by doing the necessary conversions: T=25+273=298K AS°=-236 J'mol "K' =-0.236 kJ'mol""K"' Substituting these values into the expression for AG®: AG® =—1854 kJ-mol ' (298 K)(~0.236 kJ-mol ' -K')=—1784kJ - mol” which is choice (A). Choice (B) is incorrect because the calculation was done without changing the temperature from °C to K. Choice (C) is incorrect because the calculation was done making a sign error in the second part of the expression. Choice (D) is incorrect because the calculation was done without converting the value of the AS® to kJ. Practice Questions Related to This: PQ-9 and PQ-10 SQ-4. ‘When NH4NO; dissolves in water, the temperature of the solution decreases. What describes the w"p‘ enthalpy and entropy changes of the system and which change drives the process? (A) AH = (~) and AS = () and the process is driven by the enthalpy change. (B) AH=(-)and AS=(+) and the process is driven by the enthalpy change. (C) AH=(+)and AS = () and the process is driven by the entropy change. (D) AH = (+) and AS = (+) and the process is driven by the entropy change. Knowledge Required: (1) Definition of endothermic and exothermic processes. (2) Prediction of entropy change for a process. (3) The sign of AG for a spontaneous process. 177
Your preview ends here
Eager to read complete document? Join bartleby learn and gain access to the full version
  • Access to all documents
  • Unlimited textbook solutions
  • 24/7 expert homework help
Thermodynamics Thinking it Through: You are asked to use an observation to determine the signs of the enthalpy and entropy change for the dissolving of NH:NO; in water. The first thing you notice is the salt dissolves so the process is spontaneous. The sign of AG for a spontaneous process is negative. You recall that endothermic processes take energy in from the surroundings, this results in a (+) sign for AH. Exothermic processes release energy into the surroundings and the sign of AH would be negative (~). The question tells you that the solution gets cold when the NH4NOs dissolves in water. You recognize that this means the process is endothermic, AH > 0 because the dissolution process is absorbing energy from the solution (part of the surroundings). You also know that when the solid NHsNO; dissolves in water the ions become hydrated: NH:NOs(s) = NHy'(aq) + NOs(aq) This results in an increase of entropy, AS >0 You recall that for a spontaneous process, the value of AG < 0 and the relationship for AG is: AG =AH-TAS. Putting these all together: AG < 0, so AH TAS < 0 and therefore, AH < TAS. Recall that, AH > 0 and AS> 0 and the process is driven by the entropy change. The correct answer is choice (D). Choice (A) is incorrect because the signs of AH and AS are incorrect and the process is being driven by the entropy change. Choice (B) is incorrect because the sign of AH is incorrect and the process is being driven by the entropy change. Choice (C) is incorrect because the sign of AS is incorrect. Practice Questions Related to This: PQ- 11, PQ-12, and PQ-13 SQ-5. What is the free energy change for the formation of one mole of ammonia from its elements under standard conditions? 2NHs(g) Na(g) + 3Ha(g) AG®=33.0 kJ'mol”! (A) ~66.0 kJ-mol™! (B) ~33.0 kJ-mol”! (€) -16.5kJ-mol”! (D) 66.0 kJ-mol! Knowledge Required: (1) How to manipulate thermochemical equations. Thinking it Through: You are being asked to find the free energy of formation for a substance from a given reaction. You recall that the definition of a formation reaction: A reaction that produces one mole of the substance from its elements in their standard states. You write the formation reaction for ammonia: %2 Na(g) +*2Ha(g) NHi(g) You now go back to the equation you were given in the question: 2NHs(g) Na(g) + 3Ha(g) AG®=33.0 kJ-mol”! You reverse the reaction, remembering to change the sign of AG®: Na(g) + 3H(g) 2NHs(g) AG®=-33.0 kJ-mol”! You recognize that this reaction produces ammonia from its elements, but it produces 2 moles of ammonia. To get the formation reaction, you need to multiply the reaction by %: V2 Na(g) + *2Ha(g) NHi(g) AG® = ¥(-33.0 kJ-mol™') = ~16.5 kJ-mol ! The correct choice is (C). Choice (A) is incorrect because to get this answer the reaction was multiplied by 2 instead of by %2. Choice (B) is incorrect because to get this answer the reaction has not been multiplied by ¥ . Choice (D) is incorrect because to get this answer the sign on AG was not inverted and was multiplied by 2 instead of V2. Practice Questions Related to This: PQ-14 and PQ-15 178
Your preview ends here
Eager to read complete document? Join bartleby learn and gain access to the full version
  • Access to all documents
  • Unlimited textbook solutions
  • 24/7 expert homework help
- ACS Thermodynamics Exams $Q-6. Calculate the standard Gibbs free energy change at 298 K for the AG°1/ kJ-mol™! reaction: CO(g) ~137.2 Fe304(s) + 4CO(g) = 3Fe(s) + 4COx(g) COx(g) -394.4 Fe;04(s) -1014 A) —757 kJ-mol™* (B) ~14.8 kJ-mol™! © 14.8 ki-mol™ (D) 757 kJ-mol! Knowledge Required: (1) Hess’s law. (2) Rules for manipulating free energy values. Thinking it Through: You recall that Hess’s law allows you to calculate the AG of a reaction using tabulated free energies of formation values. This works because we can imagine any reaction as consisting of first converting the reactants back into their elements and then forming the products from these elements. This is, in effect reversing the formation reactions of the reactants and then using the formation reactions of the products. You recall that this process is represented in the relationship: AG’, = 3" nAG! (products) ) mAG? (reactants) You recall that the symbol 3. means to sum and 7 and m are the stoichiometric coefficients of the reactants and products. You also notice that there is no standard free energy of formation for Fe(s) given, this is because you know the standard free energy of formation of an element in its standard state is zero. The value of AG°x, can be calculated as: G, =[4AG}(CO. (@) + 3AG; (Fe(s)) |-[4AG; (CO(@)) + AG; (Fe,0,(s)) | . ~394 kJ 0kJ —137.2 kJ —~1014kJ AG’, =| 4 mol CO, | ————— |+3 mol Fe ~| 4 mol CO| ——— | +1 mol Fe,0, | ——— mol CO, mol Fe mol CO . mol Fe O, AG;, =-148k)-mol” Choice (B) is the correct choice. Choice (A) is incorrect because it did not use the coefficients in the balanced equation and used reactants minus products. Choice (C) is incorrect because it calculated the answer using reactants minus products. Choice (D) is incorrect because it did not use the coefficients in the balanced equation. Practice Questions Related to This: PQ-16, PQ-17, PQ-18, and PQ-19 SQ-7. "An equilibrium mixture of Lx(g), Cla(g), and ICI(g) at 298 K has partial pressures of 0.0100 atm, 0.0100 atm, and 0.0900 atm, respectively. What is AG° at 298 K for the reaction: L(g) + Ch(g) = 2ICl(g) (A) -169kJ'mol! (B) -10.9 kJmol! © 109 Kl-mol! D) 169 ki mol” Knowledge Required: (1) Relationship between equilibrium and free energy. (2) How to write an equilibrium expression for a reaction. (3) How to calculate the value of K. Thinking it Through: You are given the equilibrium pressures of a system at equilibrium and need to calculate the value of AG®. You recall the relationship between free energy and equilibrium, AG = AG® + RTInQ where 0 is the reaction quotient. At equilibrium the value of AG is zero and 0 = K The relationship becomes: AG® =-RTInK You know you can calculate the value of K from the given information using the equilibrium expression: 179
Your preview ends here
Eager to read complete document? Join bartleby learn and gain access to the full version
  • Access to all documents
  • Unlimited textbook solutions
  • 24/7 expert homework help
Thermodynamics *ACS () + Clg) =2iClg) K=—ho) (0007, (P )(E) (0.0100)0.0100) AG® =-RTInK =—~(8.314 J-mol™'-K~")(298K) In(81) =—10851 J-mol"! Converting to kJ this value becomes ~10.9 kJ-mol™'. This is choice (B). You also know you can check your answer to see if it is reasonable because you have more products than reactants, so you predict a larger value of K (K> 1) and this would then be considered a spontaneous process, so you would expect a negative free energy (AG <0) as you calculated. Choice (A) is not correct because the value of K was calculated without the exponent for ICI. Choice (C) is not correct because the negative sign was dropped and the value of K was calculated without the exponent for ICI. Choice (D) is not correct because the negative sign was dropped. Practice Questions Related to This: PQ-20, PQ-21, PQ-22, PQ-23, and PQ-24 SQ-8. ‘Which statement is correct for the reaction below under standard conditions? 2Clx(g) + 2NO(g) Na(g) + 2CLO(g) AH® =-22.0 kJ-mol™ (A) The reaction is spontaneous at all temperatures. (B) The reaction is spontaneous only at low temperatures. (©) The reaction is spontaneous only at high temperatures. (D) The reaction is not spontaneous at any temperature. Knowledge Required: (1) Relationship between AG, AH, AS, and 7. (2) Definition of spontaneous. Thinking it Through: You are being asked to predict the spontaneity of a chemical reaction. You notice that you are only given a value of AH° and not for AS®. You also remember that a reaction will be spontaneous under standard conditions when AG® is negative. You also recall the relationship for AG°, AH®, AS°, and T. AG®=AH® - TAS°. The sign of AG° depends on the signs and magnitudes of AH° and AS® and the value of the temperature. These relationships are summarized below: AH° | AS° | -TAS° | AG° | Result - + - - Spontaneous at all temperatures + - + + Nonspontaneous at all temperatures -~ - + +or —_| Spontaneous at low temperature/nonspontaneous at high temperature + + - +or— | Spontaneous at high temperature/nonspontaneous at low temperature You know the sign of AH® is negative. You need to determine the sign of AS®. Recalling what you know about molar entropy values, you notice that the reaction produces 3 moles of gas from 4 moles of gas. You predict that the reaction will have a negative AS°. The table tells you that this type of reaction will be spontaneous at low temperature. This is choice (B) and the other choices are incorrect. Practice Questions Related to This: PQ-25, PQ-26, and PQ-27 SQ-9. For a reaction where K > 1 at all temperatures, which L AH? is negative statement(s) must be true? 1. AS® is positive L. AG© is positive (A) Onlyl (B) Onlylll (C) Bothlandll (D) Bothland Il 180
Your preview ends here
Eager to read complete document? Join bartleby learn and gain access to the full version
  • Access to all documents
  • Unlimited textbook solutions
  • 24/7 expert homework help
.>ACS Thermodynamics & Exams f{fiowledge Requiréd: (1)A}i'elafil;i'm;s'};ip between 'AGM,AAH. AS and T. (Z)Vkeylalionship between AG and K. Thinking it Through: You are told that the reaction has a value of K greater than 1 at all temperatures. The relationship between AG®and K is: AG"=-RTlmK or K=¢ When AG? is a negative number the value of K is always greater than 1, and when AG® is positive, the value K is less than one. Because the value of K > 1 at all temperatures, the value of AG® must be negative. This can happen at all temperatures when AH® is negative AND AS? is positive. See the table in SQ-8 for ali the possible combinations. The correct choice is (C). Choice (A) is incorrect because a negative AH® is not sufficient to make a reaction spontaneous at all temperatures. Choice (B) and (D) are not correct because if AG® were positive. K would be less than one at all temperatures. Practice Questions Related to This: PQ-28, PQ-29, and PQ-30 Practice Questions (PQ) M,\a‘ PQ-1. Which gas has the largest molar entropy at 298 K and 1 atm? (A) Ar (B) CsHs (©) CO, (D) HCI A PQ-2. Which best explains why the entropy of a sample of water increases as the temperature changes from M 25°C1075 °C? (A) There is more hydrogen bonding at the higher temperature. (B) More possible energy states are available at the higher temperature. (C) There are fewer possible ways to distribute energy at the higher temperature. (D) More water molecules can be broken into hydrogen and oxygen at the higher temperature. M PQ-3. In which reaction will the entropy of the system increase? (A) Na(g) +3H:(g) 2NHs(g) (B) 2Mg(s) = Oa(g) 2MgO(s) (C) CaCOs(s) CaO(s) ~ COx(g) (D) H0(1) + COx(g) H2COx(aq) M\ PQ-4. Which process is associated with a negative entropy change (AS®w, < 0)? (A) NaCl(s) Na*(aq) + Ci(aq) (B) Ni(g)~ 3Hx(2) 2NHs(g) (©) 2BrCl(l) Brx(D) + Cla(g) (D) H20(1) H:20(g) o PQ-5. For which of these processes is the value of AS expected to be negative? vfl"‘ I Sugar is dissolved in water. 1L Steam is condensed. 1L CaCOs is decomposed in to CaO and CO,. (A) Tonly (B) ¥and Ik only (C) Monly (D) 1land Nl only s PQ-6. For which process is the entropy change per mole the largest at constant temperature? (&) H0() Ha0(g) (B} Hx0(s) H:0(g) (C) H:0(s) H20() (D) H0(l) H20(s) 181
Your preview ends here
Eager to read complete document? Join bartleby learn and gain access to the full version
  • Access to all documents
  • Unlimited textbook solutions
  • 24/7 expert homework help
Thermodynamics ."ACS Mp\ PQ-7. In which process is entropy decreased? (A) dissolving sugar in water (B) expanding a gas (C) evaporating a liquid (D) freezing water PQ-8. Which figure represents a process with a positive entropy change? o © ° e o o 05 © © 9 o —» @w| o © ®|® o © To o L L] e & & ¢ ®o g © ?&%’ - &cc’;og ®) ‘56’\ —> ?d% PQ-9. For the reaction: 2PCIs(l) + Ox(g) 20PCls(g) AH®=-508 kJ'mol~' and AS®=—178 J-mol"K~" at 298 K. What is AG® for this reaction? (A) -561 kJ-mol’ (B) —504 kJ'mol'’ (©) 455 kJ'mol”! (D) 52,500 kJ-mol™! PQ-10. What is AG°na for 2CO(g) + Ox(g) 2COs(g) if AH % = —566 kI mol™! and AS°n, =—173 J-mol-K-! at298 K. (A) 319K mol ! (B) -514kJ-mol’’ (©) 739 kl'mol! (D) 5099 kImol”! PQ-11. What are the signs of the entropy change and enthalpy change for the combustion reaction of magnesium? 2Mg(s) + Oz(g) 2MgO(s) (A) AH>0,A5>0 (B) AH>0,AS<0 (C) AH<0,AS5>0 D) AH<0,AS<0 w‘f\ PQ-12. Which of the relationships are true about water boiling in a container that is open to the atmosphere? (A) AH>0,AS>0 (B) AH>0,AS<0 (C) AH<0,AS>0 (D) AH<0,AS<0 PQ-13. What are the signs of A// and AS for the deposition of Zn? AH AS (A) positive positive (B) positive negative (C) negative positive (D) negative negative PQ-14. Given: C3Hi(g) + 504(g) 3COx(g) + 4H:0(2) AG®nn=-2074 kJ-mol ' What is the value of AG°xn for the reaction: 6COs(g) + 8H,0(g) 2C;Hs(g) + 100a(g) (A) 1037 kJ-mol™! (B) 1037 kJ-mol”! (C) 2074 kJ'mol ! (D) 4148 kJ-mol™ 182
Your preview ends here
Eager to read complete document? Join bartleby learn and gain access to the full version
  • Access to all documents
  • Unlimited textbook solutions
  • 24/7 expert homework help
g ACS Thermodynamics Exams PQ-15. What is AG® for the reaction 2N:0(g) + 30s(g) 4NOs(g)? 4NO(g) 2N:0(g) + Ox(g) AG°=-139.56 kJ'mol”! 2NO(g) + Oa(g) »2NOa(g) AG°==69.70 ki-mol! (A) 0.16 kJ-mol™ (B) 2.09 kJ-mol™! (C) 69.86 kJ-mol™ (D) —209.26 kJ-mol™! PQ-16. Which has a value of zero at 25 °C? (A) AG? of O(g) (B) AG®r of Oa(g) (C) AG® of Os(g) (D) all of these PQ-17. What is the value of AG°x for the reaction below? AG®t/ kJ-mol! 4PCly(g) 6Clx(g) + Pa(g) PCli(g) ~269.6 Py(g; 24.1 (A) 1103 kJ-mol™ (B) -1054 kJ-mol! (C) -321kJ'mol™ (D) 1103 kJ-mol™ PQ-18. What is the value AG® for this reaction? AG®t/ kJ-mol™! COy(g) + 2H20(g) CHa(g) +20:(g) COxg) 394 H:0(g) -229 CHai(g) ~50.8 (A) 801 ki'mol™! (B) -118 kJ:mol™ (C) 572KkJ'mol™! (D) 801 kJ'mol! PQ-19. What is always true for a spontaneous process at constant pressure? (A) AGggem <0 (B) ASgsen >0 (C) ASiysien > 0 and AGsysiem >0 (D) ASiysem >0 and AGiysem =0 PQ-20. Consider the ionization of nitrous acid at 25 °C: HNO:(aq) + H:0(1) = H;0%(aq) + NOx(aq) ~ AG°=19.1 kI'mol™ ‘What is the value of K for this equilibrium? (A) 20x107 (B) 45x10+ © 22 ® 22x10° PQ-21. The ionization of aqueous hydrofluoric acid has an equilibrium constant 6.31x10* at 25 °C. What is the value of AG®n, for this reaction? (A) 0.665 kJ-mol™ ! (B) 1.53kJ-mol™ (©) 7.93 kI'mol”! (D) 18.3 ki-mol™ PQ-22. For the generic reaction shown, K = 0.355 (at 950 K). What can you conclude about AG and AG® in a reaction mixture for which Q = 0.205 at 950 K? 2AZ:(g) + Za(g) = 2AZ:(g) (A) AG and AG® are both negative (B) AG and AG® are both positive (C) AG is positive and AG® is negative (D) AG is negative and AG® is positive PQ-23. What is the value of the equilibrium constant X for a reaction for which AG® is equal to 5.20 kJ-mol™ at 50 °C? (A) 0.144 (B) 0287 ©) 693 (D) 86.4 PQ-24. For the reaction NH.CI(s) = NHs(g) + HCI(g) AH® =176 kJmol™ and AG® =91.2 kJ'mol™' at 298 K. What is the value of AG at 1000 K? (A) -109 kJ'mol"! (B) 64 kI-mol! (©) 64Kk mol” () 109 ki'mol”! 183
Your preview ends here
Eager to read complete document? Join bartleby learn and gain access to the full version
  • Access to all documents
  • Unlimited textbook solutions
  • 24/7 expert homework help
Thermodynamics ."ACS w’v‘,\d PQ-25. Thereactionis Fe:05(s) + 3C(s) 2Fe(s) + 3CO(g) AH® =+490.7 kJ-mol”! AS®=+541 J-mol K- (A) never spontaneous. (B) always spontaneous. (C) spontaneous only below a certain temperature. (D) spontaneous only above a certain temperature. v‘”,p\ PQ-26. A spontancous reaction has a negative AS. What must be true about the reaction? (A) The reaction is exothermic and the temperature is sufficiently low. (B) The reaction is exothermic and the temperature is sufficiently high. (C) The reaction is endothermic and the temperature is sufficiently low. (D) The reaction is endothermic and the temperature is sufficiently high. A PQ-27. For the process Ox(g) 20(g), AH° = 498 kI-mol~'. What would be the predicted sign of AS®y, and o\@"’fl the conditions under which this reaction would be spontaneous? AS°n Spontaneous (A) positive at low temperatures only (B) positive at high temperatures only (C) negative at high temperatures only (D) negative at low temperatures only “‘,\ PQ-28. Which gcid h_ss amore positive value of AG® for its ionization in water, Acid Ko c,#’ and which acid is stronger? HNO, 3x10* HCN 5x10710 more positive AG® stronger acid (A) HNO, HCN (B) HNO:; HNO: (C) HCN HCN (D) HCN HNO: f" PQ-29. Given that the AG® value for a particular reaction, A = B, is negative, what can be said about the o relative portions of reactants and products when the reaction reaches equilibrium? (A) There are fewer reactant molecules than product molecules. (B) There are equal numbers of reactant and product molecules. (C) There are more reactant molecules than product molecules. (D) A relationship cannot be drawn without calculating K. and AG®. M PQ-30. When the reaction A = B reaches equilibrium, which quantity equals zero? (A) The concentrations of A and B. (B) The enthalpy change, AH. (C) The equilibrium constant, K. (D) The free energy change AG. 184
Your preview ends here
Eager to read complete document? Join bartleby learn and gain access to the full version
  • Access to all documents
  • Unlimited textbook solutions
  • 24/7 expert homework help
W= 0 PREIAAN SR WS EOATUROAOWAT® > ACS Thermodynamics @ Exams Answers to Study Questions 4.D 7. B 5 C 8. B 6. B 9.C Answers to Practice Questions 11.D 21.D 12.A 22.D 13.D 23.A 14.D 24.A 15.A 25.D 16.B 26.A 17.D 27.B 18.D 28.D 19.A 29.A 20.B 30.D 185
Your preview ends here
Eager to read complete document? Join bartleby learn and gain access to the full version
  • Access to all documents
  • Unlimited textbook solutions
  • 24/7 expert homework help
Thermodynamics 186
Your preview ends here
Eager to read complete document? Join bartleby learn and gain access to the full version
  • Access to all documents
  • Unlimited textbook solutions
  • 24/7 expert homework help
Chapter 15 Electrochemistry Chapter Summary: This chapter will focus on concepts and reactions that involve the transfer of electrons. Specific topics covered in this chapter are: Galvanic cells and cell diagrams Calculating cell potentials for galvanic reactions Relating values of £%e and AG®, and Keq Using the Nernst equation Calculating quantities in electrolytic cells Calculations using concentration cells Previous material that is relevant to your understanding of questions in this chapter include: « Significant figures (Toolbox) e Scientific notation (Toolbox) * Balancing redox reactions (Chapter 5) * Definitions of anode, cathode, reduction, and oxidation (Chapter 5) « Equilibrium (Chapter 11) * Thermodynamics (Chapter 14) Common representations used in questions related to this material: Name Example Used in questions related to Cell notation Ni(s) | Ni**(aq) || Ag*(aq) | Ag(s Galvanic cells Cell diagrams Redox reactions Fe metal 1.0M Sn(NO3):(aq) 10M Fe(NOs)s(aq) Where to find this in your textbook: The material in this chapter typically aligns to “Electrochemistry™ in your textbook. The name of your chapter may vary. Practice exam: There are practice exam questions aligned to the material in this chapter. Because there are a limited number of questions on the practice exam, a review of the breadth of the material in this chapter is advised in preparation for your exam. How this fits into the big picture: The material in this chapter aligns to the Big Idea of Energy and Thermodynamics (6) and Equilibrium (8) as listed on page 12 of this study guide. 187
Your preview ends here
Eager to read complete document? Join bartleby learn and gain access to the full version
  • Access to all documents
  • Unlimited textbook solutions
  • 24/7 expert homework help
Electrochemistry cACS W Exams Study Questions (SQ) SQ-1. What is the coefficient for the nitrite ion (NO,") when the redox reaction is balanced in basic solution? MnOs (aq) + NO;(aq) MnOx(s) + NOs(aq) @ 1 ®) 2 © 3 o 4 Knowledge Required: (1) Ability to write half-reactions. (2) Rules for balancing redox equations. Thinking it Through: Y ou are given a redox reaction and are asked to determine the coefficient of a species when the equation is balanced in basic solution. You recall that the rules for balancing in acidic and basic solution are the same until the second to last step (where you balance charge using H for acidic solutions and OH~ for basic solutions). The first step is to separate the reaction into two half-reactions and proceed as shown in the table for each half reaction: Step Rxn 1 Rxn2 Asi;m E 45 47 " ?s’:o:,"‘f:bi'?‘gf NO; (ag) NO; (aq) MnO; (aq) - MnO, (s) . Oxidation reduction Assignas ox/red (2oing from +3 to +5 is a loss of electrons) (going from +7 to +4 is a gain of electrons) Balance element 3o S 7 4 undergoing NO; (aq) > NO; (aq) MnO; (aq) > MnO, (s) ox/red N already halanced Mn already balanced ‘Add e /balance 5 45 B M ox stae Sw"h NO; (aq) = NOj (aq)+2e MnOj; (aq)+3e” MnO, (s) Balance charg winot | 20H"(aq)+NO; (aq) > NO; (aq) +2¢ | MnO; (aq)+3¢” —> MnO, (aq)+ 4OH (aq) (charge below)" ' N - - Balance Hwitn | 2OH (aa)+NO;3 (ag) 2H,0(1)+MnO; (ag)+3¢” H0 NO; (aq) +2e” +H,0(1) MnO, (aq)+40H" (aq) *As a reminder, you know you would use H* if this was in acidic solution rather than basic solution. The next step is to multiple the half-reactions so that the number of electrons produced equals the number of electrons used. The overall equation is then: Ox: [20H"(aq)+NO3 (aq) > NO; (aq) + 2¢" + H,0(1)|x3 Red: [2H,0(1)+MnO; (ag)+3e MnO, (aq)+ 4OH" (aq) 2 Overall: 3NO; (aq)+2MnOj (aq) + H,0(1) - 3NO; (aq)+ 2MnO, (aq) + 20H™ (ag) Where the electrons are balanced and canceled by multiplying each (or both as here) half reaction by a factor. Finally, you canceled the extra water molecules (from the right or products side) and the extra hydroxide ions (from the left or reactants side). You can now find the coefficient for the nitrite ion (listed first) which is 3. Choice (C) is the correct answer. Choice (A) is incorrect because it is the result of not multiplying the half-reactions to make the number of electrons cancel. Choice (B) and (D) are incorrect because they are the result of incorrectly multiplying the half-reactions. Practice Questions Related to This: PQ-1, PQ-2, PQ-3 188
Your preview ends here
Eager to read complete document? Join bartleby learn and gain access to the full version
  • Access to all documents
  • Unlimited textbook solutions
  • 24/7 expert homework help
>ACS Electrochemistry @ Exams SQ-2 A spontaneous electrochemical cell is set up as shown. Which statement is true? Noltmeter Salt Bridge Sn**+2¢— Sn E°=-0.136V Fe* +3e—> Fe E°=-0036V b metal ] Sn(NO3): (aq) (A) The tin electrode is the cathode and electrons move from left to right. (B) The tin electrode is the cathode and electrons move from right to left. (C) The tin electrode is the anode and electrons move from left to right. (D) The tin electrode is the anode and electrons move from right to left. Knowledge Required: (1) Definition of spmfuc{aous reaction. (2) Definition of anode and cathode. Thinking it Through: You are being asked to evaluate some statements about a spontaneous electrochemical cell. You begin by using the given cell potentials to determine the spontaneous reaction. Recall that for a reaction to be spontaneous the value of is positive because reactions with positive cell potentials have negative free energy changes, and are spontaneous. To get the spontaneous reaction you need to reverse the reaction involving Sn. You also have to multiply the half-reactions so that electrons cancel (here 6 electrons are being transferred as written). Remember that the oxidation half-reaction provides the electrons for the reduction half-reaction. You also remember to NOT multiply the values of since these cell potentials are intensive properties and so their value does not depend on the number of times the half-reaction occurs. Sn Sn?" +2e” E°=0136V oxidation Fe* +3¢” Fe E°=-0.036 V reduction 3(Sn Sn** +2¢) E°=0.136V 2(Fe*" + 3¢~ Fe) E°=-0.036 V 3Sn + 2Fe™ 38n** + 2Fe E°=0.100 V You also know that the oxidation reaction occurs at the anode and the reduction occurs at the cathode. For the spontaneous reaction, the anode is the Sn and the Fe is the cathode. You know that electrons will move from where they are produced (the oxidation half-reaction or the reaction at anode) to where they are consumed (the reduction half-reaction or the reaction at the cathode). The source of electrons for the reduction is the anode. Therefore, the correct choice is (C). Choices (A) and (B) are not correct because tin is the anode. Choice (D) is not correct because the electrons move from the anode to the cathode. Practice Questions Related to This: PQ-4, PQ-5, PQ-6 SQ-3. Given the standard reduction potentials at 25 °C, Half Reaction what is the standard cell potential of a Aflg'(aq) +e Ag(.s) 079V Ni(s) | Ni?*(aq) || Ag'(aq) | Ag(s) galvanic cell? Ni**(aq) + 2¢” Ni(s) 023V (A) 0.56 V (B) 102V ©) 135V (D) 181V 189
Your preview ends here
Eager to read complete document? Join bartleby learn and gain access to the full version
  • Access to all documents
  • Unlimited textbook solutions
  • 24/7 expert homework help
Electrochemistry . ACS W Exams | Knowledge Required: (1) Ability to interpret electrochemical cell notation. (2) Definition of a galvanic cell. Thinking it Through: You are told the cell is galvanic and you remember that galvanic cells are spontaneous reactions. For a reaction to be spontaneous under standard conditions the value of AG° must be negative. You also recall that when AG® is negative, then £°..s positive. You are given the reaction in electrochemical cell notation. You know that in this notation, the first pair represents the reaction at the anode (oxidation) and the second pair is the reaction at the cathode (reduction). The double lines (*}|) represent the salt bridge. oxidation half-reaction||reduction half-reaction oxidation reactant | oxidation product || reduction reactant | reduction product Converting the given notation Ni(s) | Ni*(aq) || Ag'(aq) | Ag(s) to half-reactions you write: Ni(s) —» Ni*"(aq) +2¢° E°=023V (Note you changed the sign of £° from the reduction in the table) Ag(aq)+¢ Ags) E°=079V Multiplying the Ag" half-reaction by 2 to make the electrons cancel, and remembering to NOT multiply the values of the cell potentials, since they are intensive properties, you get. Ni(s) + 2Ag*(aq) 2Ag(s) + Ni**(aq) E°=102V whichis choice (B). Choice (A) is not correct because the half-reaction for Ni was not reversed. Choice (C) is not correct because the half-reaction reaction for silver was multiplied by 2 and the Ni reaction was not reversed. Choice (D) is not correct because the half-reaction reaction for silver was multiplied by 2 and the Ni reaction was reversed. Practice Questions Related to This: PQ-7, PQ-8, PQ-9, PQ-10, PQ-11 SQ-4. An oxidation-reduction reaction in which 3 electrons are transferred has a AG° = 18.55 kI-mol~* at 25°C. What is the value of £°7 A 0192V B 0064V © 0192V D) 0577V [Knowledge Required: (1) The relationship of fiee encrgy to cell potential. Thinking it Through: You are given a value of AG® and are asked for the cell potential. You recall the relationship between AG® and £°1 AG® =-nFE° [Note: It isn’t needed in this problem but the value of £° is also related to the equilibrium constant.} AG°=-RTInK so -RTInK =-nFE® E°=(RT/nF)ink where » is the number of electrons transferred and F is the Faraday constant. Note: The value of the Faraday constant will typically be given on the reference page of the exam with other constants. Solving for you get: E°=-AG/nF Substituting values results in: 1000) —(13.55kJ)(——~) 1kJ 96485 C (3 mol electrons)| - ———————- 1 mol electrons ) 0.06421-C”' =~ 0.0642V Where you have used the definition 1 V=1 J-C"'. The correct choice is (B). Choice (A) is not correct because it omitted the number of electrons and dropped the negative sign. Choice (C) is not correct because it omitted the number of electrons. Choice (D) is not correct because it multiplied by the number of electrons rather than dividing. Practice Questions Related to This: PQ-12, PQ-13 190
Your preview ends here
Eager to read complete document? Join bartleby learn and gain access to the full version
  • Access to all documents
  • Unlimited textbook solutions
  • 24/7 expert homework help
Electrochemistry Consider the reaction. Cu?(aq) + Fe(s) Cu(s) + Fe*(aq) E°=0.78V What is the value of E when [Cu?"] is equal to 0.040 M and [Fe?] is equal to 0.40 M? (A) 072V (B) 075V (©) 081V (D) 0.84V Knowledge Required: (1) Use the Nemst equation to calculate cell potential under nonstandard conditions. Thinking it Through: You recall the relationship between £ and £° is given by the Nernst equation. [Note: the equation will be provided either in the problem or on the reference sheet.] There are two common versions of the Nernst equation: og O The 0.0592 comes from using the values for R, F, and 25 °C (298 K) and the conversion between the natural and base-10 log. You then use the reaction to write an expression for Q; recall from chapter 11 that Q is an expression written exactly like an equilibrium expression, but the values substituted into the expression are not necessarily equilibrium values. Once the expression is written and the appropriate values are used you calculate Q. RT E=E"~—1 nF nQ Reactants Cn:'] T 00a0M Now, using the value of n =2 because 2 mole of electrons are transferred for every mole of reaction: 0.0592 E=078V- 1og(10)=0.75V which is choice (B). Choice (A) is incorrect because it omits n. Choice (C) is incorrect because it added the nonstandard cell potential term, (0.0592/2)logQ. Choice (D) is incorrect because it added the nonstandard cell potential term and didn’t use n =2. Practice Questions Related to This: PQ-14, PQ-15, PQ-16, PQ-17, PQ-18, PQ-19, PQ-20 $Q-6. ‘What half-reaction occurs at the anode during the electrolysis of molten sodium iodide? A) A —bL+2e B) L+2e 20 © Na—Na"+e" (D) Na"+e —Na Knowledge Required: (1) The definition of electrolysis. (2) Definitions of anode and cathode in electrochemical cells. Thinking it Through: You are being asked to identify the reaction occurring at the anode of an electrolytic cell. You remember that electrolysis reactions are reactions that use electrical energy to drive a chemical process. Specifically, these reactions have a positive AG and can be driven by the application of electrical energy. This is different from the spontaneous reactions used in galvanic cells, which have negative AG values and the excess free energy can be used to do electrical work. You also remember that no matter if the cell is a galvanic or electrolytic cell, oxidation occurs at the anode and reduction occurs at the cathode. Because the electrolysis involves molten sodium iodide, you do not have to consider the oxidation or reduction of water. Remember that in aqueous solutions, these reactions are often the ones that occur. Recall that in molten salts, the species of interest are the ions. For molten sodium iodide the ions are: Na* and [". These are the only options for reactants for the half-reactions. Because the process at the anode will be oxidation, you eliminate Na“ because it is in its highest oxidation state. Therefore, the only possible reaction is the oxidation of I: 21" I, + 2e~ which is choice (A). Choice (B) is not correct because this is the reduction of I,. Choice (C) is not correct because it is the oxidation of Na, a species not present in the molten salt. Choice (D) is not correct because it is the reduction of Na'. Practice Questions Related to This: PQ-21, PQ-22, PQ-23 191
Your preview ends here
Eager to read complete document? Join bartleby learn and gain access to the full version
  • Access to all documents
  • Unlimited textbook solutions
  • 24/7 expert homework help
Electrochemistry > ACS = W Exams How many minutes are required to plate 2.08 g of copper at a Constants constant current flow of 1.26 A? Molar mass, Cu | 63.5 g'mol' Cu(aq) + 2&” Cu(s) 1 faraday (F) 96485 C (A) 41.8min (B) 83.6 min 133 min ®) 5016 min Km)wlzdée Req;:ired: 1) The stoichiometry of electrolytic processes. i2) The relat}onships among an"\i)eres: faradays, time, and moles of electrons. Thinking it Through: You are given a mass and current. You recall that an Ampere is defined as 1 C of charge per second: 1A= l_ . Therefore, the 1.26 A can be written as 1.26 C's™ or as a conversion factor; 1.26 C=1s. s The stoichiometric relationship involving electrons and copper is: 2 mol ¢™ = 1 mol Cu The Faraday is also a conversation between coulomb and moles of electrons: 1 mol e” = 96485 C You can use these relationships to find the time required. You start with the mass of copper desired: 1 mol Cu 2mole” {96485 C 1s 1 min N L B 2.08 g Cu =836 min which is choice (B). 63.5gCu/\ 1 mol Cu /\1mole” /\1.26 C/\ 60s Choice (A) is incorrect because it didn’t use the 2 mol of electrons per 1 mol Cu relationship. Choice (C) is incorrect because it incorrectly multiplied by 1.26. Choice (D) is incorrect because it is the number of seconds, and did not convert the time to minutes. Practice Questions Related to This: PQ-24, PQ-25, PQ-26, PQ-27, PQ-28 SQ-8. For the concentration cell Fe(s) | Fe*(aq), 0.100 M || Fe**(aq), 1.00 M | Fe(s) what is Ecar at 25 °C (in mV)? (A) 9.87 mV/ (B) 19.7 mV/ ©) 29.6 mV D) 59.2mV Knowledge Required: (1) Definition of a concentration cell. (2) Ability to interpret electrochemical cell notation. | Thinking it Through: You are being asked about a concentration cell. You recall that in a concentration cell the source of the electrical potential, and the current flow, is the differences in the concentrations of the species. The given cell notation can be converted to half-reactions: Fe(s) | Fe**(aq), 0.100 M || Fe¥(aq), 1.00 M | Fe(s) oxidation / \lmduction Fe(s) Fe¥*(aq, 0.100 M) + 3e~ Fe¥*(aq, 1.00 M) + 3¢~ Fe(s) After adding the half-reactions, you get: Fe(s) + Fe**(aq, 1.00 M) Fe**(aqg, 0.100 M) + Fe(s) The Nernst equation takes into account the dependence of cell potential on concentrations: 5 [0.0592 o = Foa Jlog O remembering that Q is the ratio of the concentrations of products to reactants. n You also recall that the E°; for a concentration cell is zero, because the two half-reactions are the same: 192
Your preview ends here
Eager to read complete document? Join bartleby learn and gain access to the full version
  • Access to all documents
  • Unlimited textbook solutions
  • 24/7 expert homework help
ACS Electrochemistry Exams 00592) (0.100 M E, = o-( jlog ~00197V 3 1.00M which is choice (B). 1000 mv 0.0197 v[ool%] =19.7 mV Choice (A) is not correct because it used 6 for the number of electrons in the Nernst equation. Choice (C) is not correct because it used 2 for the number of electrons in the Nernst equation. Choice (D) is not correct because it did not use the number of electrons in the Nernst equation. Practice Questions Related to This: PQ-29, PQ-30 Practice Questions (PQ) PQ-1. What is the coefficient of the hydroxide ion for the half reaction when the hypobromite ion reacts to form the bromide ion in basic solution? BrO(aq) > Bri(aq) unbalanced )1 (B) 2 © 3 D) 4 PQ-2. What is the balanced half reaction of the nitrate ion reacting to form nitrogen monoxide in acidic M solution? (A) NOs™(aq) = HoO(l) > NO(g) + 3e” + 2H (aq) (B) NOy(aq) +3¢” ~ 4H'(aq) > NO(g) + 2H:0(1) (C) NOs(aq) + 3¢+ 2H0(l) NO(g) + 40H(aq) (D) NOs(aq) + 4™~ 5H'(aq) = NO(g) + 2H,0(1) PQ-3. What is the coefficient for water when the reaction is balanced in acidic solution? Cr;07*(aq) + Mn**(aq) - Cr*(aq) + MnOx(s) unbalanced @A) 1 (B) 2 ©) 4 @) 5 PQ-4. Which reaction occurs at the cathode of a galvanic cell constructed from these two half-ceils? The standard reduction potential of gold(III) is +1.50 V. The standard reduction potential of tin(II) is -0.14 V. (A) Au*(ag) = 3e”— Au(s) (B) Sn**(aq) +2¢” Sn(s) (©) Au(s) Au**(ag) + (D) Sn(s) Sn**(aq) + 2 PQ-5. Which reaction will occur if each substance is Standard Reduction Potentials in its standard state? Ni*(aq) +2e” Ni(s) -028 V Assume potentials are given in water at 25 °C. Sn*(aq) + 2e” Sn**(aq) 0.15V Bry(l) + 2e” 2Br7(aq) 1.06 V (A) Ni** will oxidize Sn** to give Sn** (B) Sn** will oxidize Br~ to give Bry (C) Br, will oxidize Ni(s) to give Ni*~ (D) Ni*" will oxidize Br; to give Br- 193
Your preview ends here
Eager to read complete document? Join bartleby learn and gain access to the full version
  • Access to all documents
  • Unlimited textbook solutions
  • 24/7 expert homework help
Electrochemistry *ACS Exams PQ-6. Which represents a spontaneous reaction and what is | Standard Reduction Potentials _E° the correct E°u? Fe'"(aq) + Fe(s) 004V Clyg) + 2¢” 2Cl(aq) 136V Reaction E%an (A) Fe(s) + Cly(g) Fe''(ag) + 2CI(aq) 140V (B) Fe¥(aq) + Cly(g) Fe(s) + 2CI(aq) 140V (C) 2Fe(s) + 3Ch(g) 2Fe**(aq) + 6Cl(ag) 140V (D) 2Fe(s) +3Cla(g) 2Fe™"(aq) + 6CI(aq) ~ 4.16 V PQ-7. Which combination of reactants will produce the Standard Reduction Potentials greatest voltage based on these standard electrode Cu'*(aq) + e~ Cu(s) 0.52V potentials? Sn*(ag) = 2¢- Sn’*(aq) 015V Cr'(aq) + e Cr*(aq) 041V (A) Cu’and Sn* (B) Cu*and Cr** (C) CuandSn* (D) Sn** and Cr** PQ-8. What is the standard electrode potential for a voltaic | Standard Reduction Potentials £E° cell constructed in the appropriate way from these two | Cr**(aq) + 3¢~ Cr(s —0.74V half-cells? Co*(aq) + 2" Cos, 028V (A) -1.02V (B) 046V (C) 064V D) 1.02V PQ-9. What is the value of the missing standard reduction Standard Reduction Potentials potential? ~ Pd?*(aq) + 2e~ Pd(s) ? Cu(s) + Pd**(aq) Cu?(aq) + Pd(s) £ = 0.650 V Cu*(aqg) + 2e”— Cu(s). 0337V, (A) 0987V (B) -0.987V (©) 0313V D) 0313V PQ-10. The standard cell potential, £°, for this reaction is 0.79 V. 61" (aq) ~ Cr.O7*(aq) + 14H'(aq) 31x(aq) ~ 2Cr'(ag) + 7H0(1) What is the standard potential for Ix(aq) being reduced to I (aq) given that the standard reduction potential for Cr,O-*<(aq) changing to Cr’~(aq) is 1.33 V2 (A) 054V (B) —0.54V 018V (D) 018V PQ-11. What i the standard cell potential, £°, for this [Standard Reduction Potentials _E° M reaction? Ce*(aq) + & Ce>*(aq) 161V Bra() +2Ce¥(aq) 2Br (aq) + 2Ce*'(aa) Bry(l) + 2 JBraq) 106V (A) 267V (B) -2.16V (€) 055V D) 267V PQ-12. Which statement is true for an electrochemical cell built from an oxidation-reduction reaction if K for the reaction is greater than 1?7 (A) AG®is negative, is negative (B) AG®isnegative, is positive (C) AG®is positive, is negative (D) AG?is positive, £° is positive PQ-13. What is the equilibrium constant at 298 K for the spontaneous | Reduction Potentials reaction that occurs when a Cu®"|Cu half-cell is connected toa | Cu?|Cu 034V Ag™ Ag half-cell? A AL 0.80 V (A) 2.8x107 (B) 6.0x10 (€) 3.6x10" 194 D) 3.7x10°®
Your preview ends here
Eager to read complete document? Join bartleby learn and gain access to the full version
  • Access to all documents
  • Unlimited textbook solutions
  • 24/7 expert homework help
.*ACS Electrochemistry W Exams PQ-14. Consider the reaction at 298 K. Sn**(aq) +2Fe**(aq) Sn*(aq) + 2Fe**(aq) E°=0617V What is the value of E when [Sn**] and [Fe**] are equal to 0.50 M and [Sn**] and [Fe**] are equal to 0.10 M? (A) 0.069V (B) 0679V (C) 0658V (D) 0576 V A PQ-15. Consider this reaction. w\w“p 2Al(s) + 6HCl(aq) ==2AICl(aq) + 3Hx(g) E°=1.66 V Which statement is true if the hydrogen ion concentration is initially at 1.0 M and the initial pressure of hydrogen gas is 1.0 atm? (A) Addition of a base should result in a value of £ which is less than 1.66 V. (B) n =3, because three moles of hydrogen is being produced. (C) E*is independent of the pH of the solution. _[mJ[acL] A [Ha) w‘"@ PQ-16. According to the Nernst equation, when Ecai = 0 then A) 0=K B) K=1 (€) AG°=0 (D) Ei=0 PQ-17. What is £ at 25 °C if [Ag'] = 0.200 M and [Cu?'] = 0.200 M? 2Ag (aq) + Cu(s) Cu*(aq) + 2Ag(s) E’e=0.460 V (A) 0419V (B) 0439V (C) 0.460V (D) 0481V PQ-18. A voltmeter connected to the cell under M standard state conditions reads +0.25 V., and the standard free energy (AG®) is —44 kJ-mol™'. Salt Bridge ) Tan If the concentration of Ni*" is increased to (e 2.0 M, the cell potential will and N AG will be than AG®. Pi(s) Ni**(aq) + 2 —Ni(s) E°=-025V (A) decrease, less negative (B) decrease, more negative (C) increase, less negative (D) increase, more negative PQ-19. What is the potential of a galvanic cell made from Standard Reduction Potentials Mn?'[Mn and Ag*|Ag half-cells at 25 °C if [Ag'] = | Mn® (aq) + 2 Mn(s) “1LI8V 0.10 M and [Mn?] = 0.10 M? Ag'(aq) + e Ag(s) 0.80 V (A) 192V (B) 195V (€) 198V D) 201V PQ-20. What is the cell potential of the voltaic zinc-copper cell at 25 °C? Zn(s)|Zn**(1.00%107° M)[[Cu?*(1.00x107%)|Cu(s) The standard cell potential of the cell is 1.10 V. (A) 092V (B) 128V © 1.01v D) 119V 195
Your preview ends here
Eager to read complete document? Join bartleby learn and gain access to the full version
  • Access to all documents
  • Unlimited textbook solutions
  • 24/7 expert homework help
Electrochemistry o ACS ¥ Exams PQ-21. During the electrolysis of an aqueous solution of CuSOx using inert electrodes M (A) the anode loses mass and the cathode gains mass. (B) the mass of the anode decreases but the mass of the cathode remains constant. (C) the mass of the anode remains the same but the cathode gains mass. (D) the anode and the cathode neither gain nor lose mass. M PQ-22. Which products are formed during the electrolysis of a concentrated aqueous solution of sodium chloride? 1Clg) 11 NaOH(aq) 1 Hy(g) (A) Tonly (B) Iand Il only (C) 1and Il only (D) LT and I wfd PQ-23. The half-reaction occurring at the anode during the electrolysis of molten sodium bromide, NaBr, is (A) Na*+e —Na (B) Na—Na"+e (C) Brz+2e—2Br (D) 2Br — Bry+2e” PQ-24. What mass of platinum could be plated on an electrode from the electrolysis of a PtNOs), solution with a current of 0.500 A for 55.0 s? The molar mass of platinum is 195.1 g-mol~. (A) 27.8mg (B) 455mg (C) 53.6mg (D) 91.0mg PQ-25. A current of 5.00 A is passed through an aqueous solution of chromium(III) nitrate for 30.0 min. How many grams of chromium metal will be deposited at the cathode? The molar mass of chromium is 52.0 gmol™!. (A) 0.027¢ ®) 162g (©) 485g ) 633g PQ-26. A vanadium electrode is oxidized electrically. If the mass of the electrode decreases by 114 mg during the passage of 650 coulombs, what is the oxidation state of the vanadium product? A -1 (B) +2 ©) +3 (D) +4 PQ-27. Given | amp of current for 1 hour, which solution would Molar mass / g'mol-1 deposit the smallest mass of metal? Fe 559 Ni_58.7 Cu 63.5 Ag 1079 (A) Fe from aqueous FeCly (B) Ni from aqueous NiCly (C) Cu from aqueous CuSO4 (D) Ag from aqueous AgNO; PQ-28. How many minutes will be required to deposit 1.00 g of chromium metal from an aqueous CrO> solution using a current of 6.00 A? (A) 186 min (B) 30.9 min (C) 15.4 min (D) 5.15 min PQ-29. What is the [Cu®"] in the oxidation half-cell if the observed cell potential is 0.068 V? Cu(s) | Cu*'(? M) | Cu?"(2.0 M) | Cu(s) (A) 0.043M (B) 0.010M (©) 0.14M D) 2.0M PQ-30. Which statement best describes a concentration cell when the solutions in each compartment are 0.50 M? (A) The cell potential is zero. (B) The cell is an electrolytic cell. (C) Anions will flow away from the anode. (D) Electrons will flow away from the cathode. 196
Your preview ends here
Eager to read complete document? Join bartleby learn and gain access to the full version
  • Access to all documents
  • Unlimited textbook solutions
  • 24/7 expert homework help
.ACS Electrochemistry we wO0 B R ol ol PErEEOONPEPER Y Exams Answers to Study Questions EELFS rE® % ww Answers to Practice Questions 11.C 12.B 13.C 14.B 15.A 16. A 17.B 18.A 19.B 20.D SRENERERERE PROBRINN AL PEITDAEPOON 197
Your preview ends here
Eager to read complete document? Join bartleby learn and gain access to the full version
  • Access to all documents
  • Unlimited textbook solutions
  • 24/7 expert homework help
Electrochemistry 198
Your preview ends here
Eager to read complete document? Join bartleby learn and gain access to the full version
  • Access to all documents
  • Unlimited textbook solutions
  • 24/7 expert homework help
Chapter 16 Nuclear Chemistry Chapter Summary: This chapter will focus on expressing and predicting nuclear reactions. Specific topics covered in this chapter are: * Radioactivity * Radioactive decay * Balancing nuclear reactions Previous material that is relevant to your understanding of questions in this chapter include: * Atomic Structure (Chapter 1) * Electronic Structure (Chapter 2) o Stoichiometry (Chapter 4) Common representations used in questions related to this material: Name Example Used in questions related to Graphs B nuclear binding energies AL T off ¢ o Binding energy per nucleon (Me' o 4 w20 w0 w0 Mass number (A) Where to find this in your textbook: The material in this chapter typically aligns to “Nuclear Chemistry” in your textbook. The name of your chapter(s) may vary. Practice exam: There are practice exam questions aligned to the material in this chapter. Because there are a limited number of questions on the practice exam, a review of the breadth of the material in this chapter is advised in preparation for your exam. How this fits into the big picture: The material in this chapter aligns to the Big Idea of Atomic Structure (1) as listed on page 12 of this study guide. Study Questions (SQ) SQ-1. What is the missing particle in the nuclear equation? ZiTh— ¥ Ra+ @ (B) iHe © H ™) n Knowledge Required: (1) How to balance a nuclear equation. Thinking it Through: You are asked in the question to determine the missing particle from a balanced nuclear equation. You can assume that all mass units are accounted for in the missing particle. Therefore, you note that the starting material (Th) undergoes the loss of 4 mass units in becoming the product shown (Ra) as 4 (remember, 4 is 199
Your preview ends here
Eager to read complete document? Join bartleby learn and gain access to the full version
  • Access to all documents
  • Unlimited textbook solutions
  • 24/7 expert homework help
Nuclear Chemistry ACS w Exams ‘mass number or the number of protons plus the number of neutrons) goes from 230 (**°Th) to 226 (***Ra). The missing particle will have a mass number of 4 (*X). In addition, you note that the starting material (Th) is 2 atomic numbers (or two protons) more than the product (Ra). as Z goes from 90 (50Th) to 88 (ssRa). The missing particle will have an atomic number of 2 (X). Four mass units with two of those as protons is a He particle. Therefore, you can conclude that this is an example of alpha decay and that ;He is the missing particle, choice (B). Choice (A) is not correct because it is the particle lost for a beta decay equation. Choice (C) is not correct because it is a proton particle. Choice (D) is not correct because it is a neutron particle. Practice Questions Related to This: PQ-1, PQ-2, PQ-3, and PQ-4 SQ2. A U atom undergoes nuclear fission by being bombarded with a neutron to produce a total of four neutrons, a '%,Cs atom, and @ U ® Np (©) ZRb (D) Rb Knowledge Required: (1) Definition of nuclear fission. (2). How to write and balance a nuclear equation. Thinking it Through: You are asked in the question to determine the missing particle for a given nuclear fission reaction. The reaction is expressed in words; it is helpful in this situation to express the reaction as an equation: BU4In o MCs+dlns Next, you balance the protons; there are 92 on the reactant side (52U and on) and 55 on the product side (s5Cs and 4 x on) or 92-55 = 37 protons missing for the unknown atom. You then balance the mass numbers; there are 235 + 1 on the reactant side (***U and 'n) and 140 + 4x1 on the product side (**'Cs and 4 * 'n) or 236 144 = 92 mass numbers missing for the unknown atom. Thus, X’ is missing in order to balance the reaction; this is the atom ? Rb, Choice (C). Choice (A) is not correct because it assumes no change in the starting material. Choice (B) is not correct because it is the result of positron emission, i.e. a neutron becomes a proton. Choice (D) is not correct because it does not account for the four neutrons that are produced in the nuclear fission reaction. Practice Questions Related to This: PQ-5 SQ-3. An atom of the element of atomic number 53 and mass number 131 undergoes beta decay. The residual atom after this change has an atomic number of and a mass number of . (A) 54,131 (B) 53,131 (C) 53,131 D) 51,127 emission occur. (2) How to write and balance a nuclear reaction. Thinking it Through: You are asked in the question to determine the atomic number and mass number for an atom after a particular radiation emission has occurred. In this instance, the atom undergoes beta decay (a particle); this emission results in a net gain of one atomic number (or transformation such that the nucleus now has one additional proton). During beta decay, there is no overall change in mass number. You identify the element in the question as iodine, I (Z= 53). i 0g 4 1 sl B+ 5Xe 200
Your preview ends here
Eager to read complete document? Join bartleby learn and gain access to the full version
  • Access to all documents
  • Unlimited textbook solutions
  • 24/7 expert homework help
ACS Nuclear Chemistry Exams Therefore, if a particle that is "1 undergoes beta decay, the resulting atom is ', Xe (Z = 54, 4 = 131), choice (A). Choice (B) is not correct because it would be the residual atom after a positron emission. Choice (C) is not correct because it would be the residual atom after a gamma emission. Choice (D) is not correct because it would be the residual atom after alpha decay. Practice Questions Related to This: PQ-6, PQ-7, PQ-8, PQ-9, PQ-10, PQ-11, and PQ-12 Practice Questions (PQ) PQ-1. Cobalt-60 undergoes beta decay. What is the balanced nuclear reaction for this process? (@A) #Co—p+5Fe ®) 5Co—— Jp+ENi #Co——ja+iMn @) #Co+ je——fFe PQ-2. What is the missing particle in the balanced nuclear equation? BMg— TNa+_ + v @ ®) B © H ®) o PQ-3. What is the missing product of this reaction? iR @ B ® B © oY ® PQ-4. What is the missing particle in the balanced nuclear equation? TAl+ jHe—> P+ 3 3 @) e ® e © H ® o PQ-5. When 2;sz captures an alpha particle, it produces a neutron and which isotope? @) “cCm ® Cf © P ® Pu M PQ-6. Which particle, if lost from the nucleus, will nof result in a change in the atomic mass number? (A) proton (B) alpha particle () beta particle (D) neutron PQ-7. An atom of the element of atomic number 84 and mass number 199 emits an alpha particle. The residual atom after this change has an atomic number of and a mass number of (A) 82,195 (B) 84,203 (C) 85,195 (D) 86,199 PQ-8. What is the product when *;Po undergoes positron emission? (@A) ‘uPb (B) Rn © D) 8Bi PQ-9. Which equation qualifies as electron capture? @ SLiton—> H+la ®) Mgt p > [Natia [Ar+ e o JClthy ® Ko Cat g 201
Your preview ends here
Eager to read complete document? Join bartleby learn and gain access to the full version
  • Access to all documents
  • Unlimited textbook solutions
  • 24/7 expert homework help
Nuclear Chemistry s A CS Exams PQ-10. Which nuclear process does not necessarily result in nuclear transmutation (the transformation of one M element into another)? (A) alpha (o) emission (B) beta (B) emission (C) gamma (y) emission (D) positron emission PQ-11. Medical imaging is used when a patient ingests a radioactive emitter, called a tracer, so a doctor can view an internal system. Which tracer emission will penetrate through the body? (A) alpha particle (B) beta particle (C) gamma particle (D) helium nucleus PQ-12. Based on their binding energies, what nuclear 5 10 transformations are predicted for F and Br? g . PN, e F Br s g4 = 8 g2 £ K A 0 4 s 120 160 200 ‘Mass number ( A) (A) Both could undergo nuclear fission. (B) Both could undergo nuclear fusion. (C) F could undergo nuclear fission and Br could undergo nuclear fusion. (D) Br could undergo nuclear fission and F could undergo nuclear fusion. 202
Your preview ends here
Eager to read complete document? Join bartleby learn and gain access to the full version
  • Access to all documents
  • Unlimited textbook solutions
  • 24/7 expert homework help
bl o= ol orww ~*ACS Nuclear Chemistry ¥ Exams Answers to Study Questions Answers to Practice Questions [adal 2k 4 crOw = gann 203
Your preview ends here
Eager to read complete document? Join bartleby learn and gain access to the full version
  • Access to all documents
  • Unlimited textbook solutions
  • 24/7 expert homework help